bridgeelizabeth bryan [email protected] bridge features this month include: 1 bidding quiz by...

48
BRIDGE Number One Hundred and Sixty April 2016 Bernard Magee’s Acol Bidding Quiz This month we are looking at slams and strong hands. The Blackwood used will be Key Card Blackwood (same responses as normal Blackwood but including the king of trumps). You are West in the auctions below, playing ‘Standard Acol’ with a weak no-trump (12-14 points) and four-card majors. N W E S 1. Dealer East. Game All. A K 9 K J 3 Q J 7 6 A 10 4 West North East South 1NT Pass ? N W E S 2. Dealer East. Game All. A J 4 3 2 3 A 5 2 Q J 7 6 West North East South 1Pass ? N W E S 3. Dealer East. Game All. A Q 4 3 4 Q J 6 3 A K Q J West North East South 1Pass ? Answers on page 8 N W E S 4. Dealer West. Love All. A K 2 K J 7 6 A 9 4 A 8 7 West North East South 1Pass 3Pass ? N W E S 5. Dealer West. Love All. K 10 4 K Q 7 6 5 2 A K Q 4 West North East South 1Pass 3Pass ? N W E S 6. Dealer West. Love All. K 8 7 6 5 4 3 Void 7 6 A K Q J West North East South 1Pass 3Pass ? Answers on page 10 N W E S 7. Dealer East. Love All. A J 4 3 J 7 6 4 3 K Q 10 3 West North East South 1Pass 3Pass 4Pass ? N W E S 8. Dealer West. Love All. A K J 8 5 J 6 3 4 K Q 7 6 West North East South 1Pass 41 Pass ? 1 Splinter bid – showing short- age in clubs and a raise to 4N W E S 9. Dealer West. Love All. A K J 8 5 Q 2 A 6 9 8 7 6 West North East South 1Pass 41 Pass ? 1 Splinter bid – showing short- age in clubs and a raise to 4Answers on page 12 N W E S 10. Dealer East. Love All. K Q 4 K Q 5 K Q J 4 K Q 5 West North East South 3Pass ? N W E S 11. Dealer East. Love All. K 9 8 2 A K 7 6 5 4 2 A 7 West North East South 3Pass ? N W E S 12. Dealer West. Love All. J 3 K Q 7 6 5 K Q 4 3 A 2 West North East South 1Pass 1Pass 2Pass 31 Pass 3NT Pass 4NT Pass ? 1 Fourth suit forcing Answers on page 14

Upload: others

Post on 14-May-2020

11 views

Category:

Documents


1 download

TRANSCRIPT

BRIDGENumber One Hundred and Sixty April 2016

Bernard Magee’s Acol Bidding QuizThis month we are looking at slams and strong hands. The Blackwood used will be Key Card Blackwood

(same responses as normal Blackwood but including the king of trumps).

You are West in the auctions below, playing ‘Standard Acol’ with a weak no-trump (12-14 points) and four-card majors.

NW E

S

1. Dealer East. Game All. ♠ A K 9 ♥ K J 3 ♦ Q J 7 6 ♣ A 10 4

West North East South 1NT Pass ?

NW E

S

2. Dealer East. Game All. ♠ A J 4 3 2 ♥ 3 ♦ A 5 2 ♣ Q J 7 6

West North East South 1♠ Pass ?

NW E

S

3. Dealer East. Game All. ♠ A Q 4 3 ♥ 4 ♦ Q J 6 3 ♣ A K Q J

West North East South 1♥ Pass ?

Answers on page 8

NW E

S

4. Dealer West. Love All. ♠ A K 2 ♥ K J 7 6 ♦ A 9 4 ♣ A 8 7

West North East South 1♥ Pass 3♥ Pass ?

NW E

S

5. Dealer West. Love All. ♠ K 10 4 ♥ K Q 7 6 5 ♦ 2 ♣ A K Q 4

West North East South 1♥ Pass 3♥ Pass ?

NW E

S

6. Dealer West. Love All. ♠ K 8 7 6 5 4 3 ♥ Void ♦ 7 6 ♣ A K Q J

West North East South 1♠ Pass 3♠ Pass ?

Answers on page 10

NW E

S

7. Dealer East. Love All. ♠ A J 4 3 ♥ J 7 ♦ 6 4 3 ♣ K Q 10 3

West North East South 1♠ Pass 3♠ Pass 4♣ Pass ?

NW E

S

8. Dealer West. Love All. ♠ A K J 8 5 ♥ J 6 3 ♦ 4 ♣ K Q 7 6

West North East South 1♠ Pass 4♣1 Pass ?

1Splinter bid – showing short-age in clubs and a raise to 4♠

NW E

S

9. Dealer West. Love All. ♠ A K J 8 5 ♥ Q 2 ♦ A 6 ♣ 9 8 7 6

West North East South 1♠ Pass 4♣1 Pass ?

1Splinter bid – showing short-age in clubs and a raise to 4♠

Answers on page 12

NW E

S

10. Dealer East. Love All. ♠ K Q 4 ♥ K Q 5 ♦ K Q J 4 ♣ K Q 5

West North East South 3♠ Pass ?

NW E

S

11. Dealer East. Love All. ♠ K 9 8 ♥ 2 ♦ A K 7 6 5 4 2 ♣ A 7

West North East South 3♠ Pass ?

NW E

S

12. Dealer West. Love All. ♠ J 3 ♥ K Q 7 6 5 ♦ K Q 4 3 ♣ A 2

West North East South 1♥ Pass 1♠ Pass 2♦ Pass 3♣1 Pass 3NT Pass 4NT Pass ? 1Fourth suit forcing

Answers on page 14

Canaries Christmas & New Year

Date Arrive Depart Destination

DECEMBER 2016

Tue 20 PM Southampton

Wed 21-Fri 23 Cruising

Sat 24 early am late night Casablanca, Morocco

Sun 25 Cruising

Mon 26 early am late pm Arrecife, Lanzarote

Mon 26 late night Puerto del Rosario, Fuerteventura

Tue 27 late night Puerto del Rosario, Fuerteventura

Wed 28 early am late night Las Palmas, Gran Canaria

Thu 29 early am late night Santa Cruz, La Palma

Fri 30 early am late night San Sebastian, La Gomera

Sat 31 early am late night Santa Cruz, Tenerife

JANUARY 2017

Sun 01 Cruising

Mon 02 early am late pm Funchal, Madeira

Tue 03 Cruising

Wed 04 early am late pm Lisbon, Portugal

Thu 05-Fri 06 Cruising

Sat 07 AM Southampton

Celebrate the festive period in the warmth and natural splendour

of the Canaries, including a lovely Christmas Day at sea. Kick back

on the pearly beaches of Fuerteventura; discover the beautiful

National Park of Garajonay in La Gomera; encounter bustling Moroccan

souks; and be sure to see in the New Year in thriving Tenerife, complete

with fireworks, live Latino music and salsa dancing.

Casablanca, MoroccoGaze in awe at the dramatic Hassan II Mosque with its 210m-high minaret, delve deep into the history of the city at the intriguing Jewish Museum, and dine in sophisticated surroundings at Rick’s Café – inspired by the mythical saloon from the legendary 1942 film, Casablanca.

Santa Cruz, La PalmaLa Palma’s pretty capital boasts a startling array of religious edifices, lush greenery and cafes that take pride in serving exquisite local cuisine. Journey to the south of the island and discover the stunning scenery and flora of the San Antonio Volcano, before learning of its history at the engaging visitor centre.

Santa Cruz, TenerifeIn Tenerife you can soak up the temperature weather on caramel-coloured beaches, and enjoy traditional Spanish Culture at New Year celebrations you’ll never forget – complete with fireworks, live Latino music and salsa dancing. Don’t miss tours to the Botanical Gardens, Pyramids of Guimar or breathtaking volcanic scenery surrounding Mount Teide – Spain’s highest Mountain.

Funchal, MadeiraFunchal has a timeless charm, delightfully enhanced by a benign climate that fills this floating garden with the year-round colours and perfumes of subtropical flowers and fruit. Explore the stunning landscapes by way of the unique levada irrigation network, admire the many beautiful gardens, or try the island’s famous wine.

Your price includes Breakfast, lunch, afternoon tea, 5-course à la carte dinner, late night buffet & self-service tea & coffee • Varied daytime activities • Lectures & talks • Swimming pools, Jacuzzis & gym • Choice of live evening entertainment • 4 formal nights • Bridge Programme

Prices are correct at time of going to print, but may change at any time. Call for latest prices.

Prices per personCabin type Prices fromInside £2,199Outside £2,499Balcony £3,999Suite £4,599Single £2,874

Contact Mr Bridge to book now on 01483 489961 or visit www.mrbridge.co.uk

Tenerife

La GomeraLa Palma

Gran Canaria

FuerteventuraLanzarote

Lisbon

Madeira Casablanca

To/from Southampton

20th December 2016 • 18 nights • Sails from Southampton • Balmoral • CRUISE ID: L1631

Terms and conditions apply - see Fred. Olsen 2016/17 worldwide brochure.

Mr Bridge Team now on board

FOC322355_MrBridge_christmasCruise_FP_Ad_W.C.18.01.16_V3.indd 1 26/01/2016 13:52

BRIDGE April 2016 Page 3

Ryden Grange, Knaphill, Surrey GU21 2TH

( 01483 489961

[email protected]

www.mrbridge.co.uk

shop: www.mrbridge.co.uk/ mrbridge-shop

Publisher and Managing Editor

Mr Bridge

Associate Editor and Bridge Consultant

Bernard Magee bernardmagee

@mrbridge.co.uk

Cartoons & Illustrations Marguerite Lihou

www.margueritelihou.co.uk

Technical Consultant Tony Gordon

Typesetting & Design Ruth Edmondson

[email protected]

Proof Readers Julian Pottage

Mike Orriel Catrina Shackleton

Richard Wheen

Customer Services Catrina Shackleton

[email protected]

Events & Cruises ( 01483 489961

Jessica Galt [email protected]

Megan Riccio [email protected]

Sophie Pierrepont [email protected]

Clubs & Charities Maggie Axtell

[email protected]

Address Changes ( 01483 485342 Elizabeth Bryan

[email protected]

BRIDGEFeatures this month include:

1 Bidding Quiz by Bernard Magee

5 Mr Bridge

8 Bidding Quiz Answers (1-3) by Bernard Magee

10 Bidding Quiz Answers (4-6) by Bernard Magee

12 Bidding Quiz Answers (7-9) by Bernard Magee

13 Avoidance by Bernard Magee

14 Bidding Quiz Answers (10-12) by Bernard Magee

15 Jacoby 2NT by Heather Dhondy

18 Catching Up with Sally Brock

19 Sally’s Slam of the Month

20 David Stevenson Answers Your Questions

23 Defence Quiz by Julian Pottage

24 Defence Quiz Answers by Julian Pottage

25 Sorry Partner! by Liz Dale

25 More Tips by Bernard Magee

26 Globe Trotter – Part Two by Shireen Mohandes

28 Trick One by Jeremy Dhondy

30 Julian Pottage Answers Your Questions

32 Declarer Play Quiz by David Huggett

33 Teachers’ Corner by Ian Dalziel

34 Declarer Play Answers by David Huggett

35 Unauthorised Information by David Stevenson

36 Cover an Honour with an Honour by Andrew Kambites

38 Robin Hood’s Awful Hand by David Bird

40 Accelerated Bridge Teaching by Ned Paul

43 Wendy Wensum’s Diaries

44 Seven Days with Sally Brock

46 Readers’ Letters

ADVERTISERS’ INDEX

2 Canaries Christmas & New Year with Fred.Olsen

3 Clive Goff ’s Stamps

4 Greece & the Greek Islands with Voyages to Antiquity

5 Mr Bridge Just Duplicate Bridge Events

6 Greece & the Greek Islands with Voyages to Antiquity

7 Better Hand Evaluation

7 Denham Filming 2017

8 QPlus 11

8 Travel Insurance

8 Designs for Bridge Tables

9 Bernard Magee’s Tutorial Software

10 Mr Bridge Playing Cards

10 Kinds of Accountant Tea Towel

10 Designs for Bridge Table Covers

11 Bernard Magee DVDs

12 Mr Bridge Tie

12 Club Insurance

12 Duplicate Bridge Rules Simplified

14 BriAn

14 Life’s a Game Tea Towel

14 Stoke Mandeville Spinal Research

16 Canaries & Madeira Winter Warmth with Fred.Olsen

17 Madeira, Canaries & Morocco with Voyages to Antiquity

19 Greece & the Adriatic Sea with Voyages to Antiquity

27 Greece & the Adriatic Sea with Voyages to Antiquity

34 Mr Bridge Tutorial Weekends

45 Irish Christmas Markets with Fred.Olsen

47 Charity Events

48 Central America & the Mardi Gras with Fred.Olsen

REDUCE THE COST OF YOUR POSTAGE

Postage stamps for sale at 90% of face-value, all mint with full gum. Quotations for

commercial quantities available on request.

Values supplied in 100s, higher values available as well as 1st and 2nd class

(eg 2nd class: 100x38p+100x16p).

(/Fax 020 8422 4906 e-mail: [email protected]

10093

ABTA No.Y2206

Join Will and Sylvia on the ultimate Greek Island cruise. Sail aboard the elegant Aegean Odyssey to picturesque islands and magnificent historic sites of the Aegean. From the ruins of Minoan Akrotiri and Knossos to sacred Delos, Patmos and Byzantine Thessalonica – this remarkable voyage features 8 included excursions and daily duplicates throughout.

GREECE AND THE GREEK ISLANDS

DELO

S

DEPARTS UK SEPTEMBER 5, 2016

FARES INCLUDE:

• Scheduled economy class flights• Expert guest speaker programme• Mr Bridge drinks party• Duplicate bridge every evening• Morning bridge seminars and afternoon

duplicates when at sea• Sightseeing excursions in most ports of call• All meals on board in choice of two restaurants• Complimentary wine with dinner on board• Gratuities for on-board cabin and restaurant staff• Overseas transfers and baggage handling

Mr Bridge fares are per person and subject to availability at time of booking. They may be withdrawn at any time without notice. Please see website for full terms and conditions. †Single accommodation is available only in certain categories and is subject to availability.

Premium Inside from £1,695pp

Standard Outside from £1,995pp

Premium Outside from £2,295pp

MR BRIDGE VALUE FARES

SEPT 5 Fly to ATHENS GreeceEmbark Aegean Odyssey in Piraeus 10.00pm

SEPT 6 HYDRA Greek Islands 7.00am 6.00pm

SEPT 7 SANTORINI Greek Islands 8.00am(Akrotiri) 6.00pm

SEPT 8 HERAKLION Crete, 7.00amGreek Islands (Knossos) 2.00pm

SEPT 9 RHODES Greek Islands 9.00am 6.00pm

SEPT 10 DELOS Greek Islands 7.00am noonMYKONOS Greek Islands 2.00pm 9.00pm

SEPT 11 PATMOS Greek Islands 7.00am 2.00pm

SEPT 12 KAVALA Greece (Philippi) 2.00pm 8.00pm

SEPT 13 THESSALONICA Greece 8.00am 7.00pm

SEPT 14 VOLOS Greece (Meteora) 7.00am 10.00pm

SEPT 15 SKIATHOS Greek Islands 7.00am 4.00pm

SEPT 16 PIRAEUS Greece 6.00amDisembark and transfer to Athens Airport for flight home

AEG160905BR

Odyssey Club Members enjoy an additional 5% discount on prices shown above. NO SINGLE

SUPPLEMENT

HOSTED BY WILL & SYLVIA PARSONS

12-day fly-cruise from £1,695 per person

NEW BRIDGE CRUISE ON BOARD AEGEAN ODYSSEY

CALL NOW ON 01483 489961OR VISIT www.mrbridge.co.ukCabins can be held at no obligation for 7 days

Rhodes

Skiathos

GreeceAegean Sea

Patmos

Athens

Kavala

Santorini

MykonosDelos

HeraklionCrete

Knossos

Hydra

PhilippiThessalonica

Volos

Meteora

2114 Mr Bridge FULL page 160905BR_Layout 1 24/02/2016 23:34 Page 1

BRIDGE April 2016 Page 5

2016 JUST DUPLICATE

BRIDGE

Full Board No Single

Supplement*

( 01483 489961 www.mrbridge.co.uk

Please note there are no seminars, set hands or prizes at these events.

*subject to availability

Chatsworth Hotel Worthing BN11 3DU

8-10 April £199

6-8 May £199

3-5 June £199

15-17 July £199

5-7 August £199

7-9 October £199

11-13 November £199

25-27 November £199

Wyndham Garden Grantham

Marston, Lincs NG32 2HT

13-15 May £189

16-18 September £189

14-16 October £189

4-6 November £189

18-20 November £189

Denham Grove Near Uxbridge, UB9 5DG

10-12 June £199

21-23 October £199

4-6 November £199

Inn on the Prom St Annes on Sea FY8 1LU

1-3 July £189

REMINDER

Just to remind you to drop me a note or its modern equivalent, an email, to enter the prize competition. Guess Grandpa’s new name. Closing date 31 March 2016.

FOR YOUR SAFETY

Voyages to Antiquity have re-routed Aegean Odyssey away from all Turkish ports. These changes make a remarketing necessary, especially the 5 September sailing advertised on the facing page.

The Med will be cooling down after the heats of high summer and there is an optional inclusive excursion almost every day.

2016 CCC

Enclosed with this magazine, is a copy of all the hands played in the 58th heats playing for the annual Charity Challenge Cup. See the results on www.ecatsbridge.com/sims It is in support of a really good cause. If you wish to contribute, please send me a cheque, payable to Stoke Mandeville Spinal Research. Meanwhile, do enjoy the commentary provided by Bernard Magee.

TEA TOWELS

We all need a round tuit, see below a reminder of other useful titles;

All printed on 100% cotton, delightfully packaged and presented. £5.00 each. Any five from our list £12.95. See the carrier sheet for our full mail order form.

JUST DUPLICATE

All the 2016 weekend events are filling up quite well. Indeed, I am pleased to see new names on the lists. Do be assured you will receive a big, warm and friendly welcome, especially if you are brave enough to come on your own. Lots of people like you do.

CLUB INSURANCE

Over 500 British bridge clubs are insured through my agency. In these litigation happy days, club committee members really need to know they are covered, not just for the obvious risks, but for those completely unforeseen events.

SECOND HAND QPLUS

One Frequently Asked Question is, What do I do with copies of QPlus sent in part exchange? Answer. I ask for a cheque in favour of Little Voice, an orphanage in Addis Ababa, Ethiopia.

Q7 suggested donation £10 Q8 suggested donation £15 Q9 suggested donation £19

Back up support is always available from Mr Bridge and the Ryden Grange team.

TRAVEL INSURANCE

Another Frequently Asked Question. If you can use the now not-so-new technology, Google ‘cover cloud’ and follow the step by step instructions. If not, give Cloud Cover a ring and a helpful soul will put your details into the template and produce a quote while you wait. ( 01268 524344 p7

Be reasonable

... do itmy way!

THE PRESENT vvv

Yesterday is HistoryTomorrow is a Mystery

Today is a GiftThat is why we call it

. . . The Present

There are Three Kinds of

Accountant . . .

Those who can add up and

those who can’t.

Life’s a GAMEbut

BRIDGEis

SERIOUS

10093

ABTA No.Y2206

Take in the Autumn sunshine as you sail around the beautifulAegean Sea. The ultimate in Greek Island cruising, this voyage includes 8 sightseeing excursions ashore to the fascinating history of Classical Greece, along with visits to some of the Aegean’s most legendary islands. On board you can enjoy expert guest speakers who will inform you of the places you will visit, and of course, duplicate bridge every evening. Call now and reserve your cabin to join Mr and Mrs Bridge on a bridge holiday you’ll never forget.

CALL NOW ON 01483 489961OR VISIT www.mrbridge.co.uk

JOIN MR & MRS BRIDGEon this 12-day Aegean Bridge Cruise

Cabins can be held at no obligation for 7 days

GREECE& THE GREEK

ISLANDS

SANT

ORIN

I

DEPARTS UK SEPTEMBER 16, 2016

Cruise fares include• Scheduled economy class flights

• Expert guest speaker programme

• Mr Bridge drinks party

• Duplicate bridge every evening

• Morning bridge seminars and afternoon duplicates when at sea

• Sightseeing excursions in most ports of call

• All meals on board in choice of two restaurants

• Complimentary wine with dinner on board

• Gratuities for on-board cabin and restaurant staff

• Overseas transfers and baggage handling

*Mr Bridge fares are per person and subject to availability at time of booking. tSingle accommodation is available only in certain categories and is subject to availability.

Please see website for full terms and conditions.

Premium Inside £1,695ppPremium Outside £2,295ppDeluxe Outside £2,595pp

MR BRIDGE VALUE FARES

Odyssey Club Members enjoy an additional discount of 5% on these fares

SEPT 16 Fly to ATHENS GreeceEmbark Aegean Odyssey in Piraeus 10.00pm

SEPT 17 HYDRA Greek Islands 7.00am 6.00pm

SEPT 18 SANTORINI Greek Islands 8.00am(Akrotiri) 6.00pm

SEPT 19 HERAKLION Crete, 7.00amGreek Islands (Knossos) 2.00pm

SEPT 20 RHODES Greek Islands 9.00am 6.00pm

SEPT 21 DELOS Greek Islands 7.00am noonMYKONOS Greek Islands 2.00pm 9.00pm

SEPT 22 PATMOS Greek Islands 7.00am 2.00pm

SEPT 23 KAVALA Greece (Philippi) 2.00pm 8.00pm

SEPT 24 THESSALONICA Greece 8.00am 7.00pm

SEPT 25 VOLOS Greece (Meteora) 7.00am 10.00pm

SEPT 26 SKIATHOS Greek Islands 7.00am 4.00pm

SEPT 27 PIRAEUS Greece 6.00amDisembark and transfer to Athens Airport for flight home

AEG160916BR

NOSINGLE

SUPPLEMENT†

Rhodes

Skiathos

GreeceAegean Sea

Patmos

Athens

Kavala

Santorini

MykonosDelos

HeraklionCrete

Knossos

Hydra

PhilippiThessalonica

Volos

Meteora

NEW ITINERARY

2114 Mr Bridge FULL page 160916BR_Layout 1 24/02/2016 23:57 Page 1

BRIDGE April 2016 Page 7

Better HandEvaluationBernard Magee

Introduction

Better Hand Evaluation is aimed at helping readers to add greater accuracy to their bidding. It deals with auc-tions in which you and your partner, against silent op-ponents, can describe your hands fully to each other and, by evaluating them ac-curately, find the best final contract. The emphasis of all good, accurate bidding is on hand evaluation.

There are two general types of auction: a) a fit is found and b) no fit is found.

When you do not have a fit, you are aiming to describe the strength of your hand as soon as possible, most often using no-trump bids. This book be-gins by discussing balanced-hand bidding in Acol, as it is very important that both members of a partnership have an accurate knowledge of how to show hands of different strengths.

When a fit is found, there is much re-evaluation of the hand to be done; point count, though still important, needs to be evaluated together with distribution. The best way of reaching an accurate assess-ment is to use the Losing Trick Count; this is an important method of hand evaluation and takes up a number of chapters.

Finally, we move on to different forms of evaluation including game tries and splinter bids. You can never know enough methods of hand evaluation; the more you learn, the better you get at judging your hand.

Although the Losing Trick Count is used more easily in tandem with your partner, a large proportion of the ideas in this book can be used by an individual. For example, eval-uating your hand to be worth an extra point is going to help anyone you partner – as long as you get it right.

£14 including UK postage

Mr Bridge ( 01483 489961 www.mrbridge.co.uk

BERNARD MAGEEat Denham Grove

near Uxbridge, Bucks, UB9 5DG.

13-15 January 2017£399pp Friday – Monday

£369pp Friday – Sunday

Full Board – No Single Supplement1

Limited places for Thursday night available. £65pp single, £40pp double/twin.

TopicsMORE SIGNALLINGI will be looking at different times when you signal and the messages you might want to give. Using signals in new ways can greatly improve your enjoyment of defence as well as pushing up your scores.

4-4-4-1 HANDSEverybody’s least favourite type of opening hand. I will be going through the methods for choosing the right suit to open as well as coping with responses. As responder you need to be aware of the options and work out your partner’s type of hand. Strong 4-4-4-1 hands can be just as difficult and will be dealt with too.

DRAWING TRUMPSThis seminar sounds straight forward, but we will not be simply drawing trumps, we will be considering the reasons for delaying. Keeping control of trumps is an important part of declarer play. Knowing when to risk leaving trumps out and when not.

FIVE-CARD MAJORSPopular around the world, this method is becoming more popular here. It is not a method I would advocate for club players, however it is important to understand the method as you will need to defend against it.

FUNDEMENTALS OF DEFENCEDefence is by far the hardest aspect of bridge: this seminar seeks to show the building blocks that can start you off on a wonderful journey. If you can get the basics right then the more complicate aspects of defence can follow.

SUPPORTING MINORSMinors as the name suggests are not as important as majors, but we have to bid them and it is important to know your system. Bidding more 3NT contracts will get you better scores, but being able to spot a minor suit slam will put you cut above.

6 seminar sessions with Bernard2

6 sessions of supervised play3

Contact Mr Bridge to book your place or for further details: ( 01483 4899611Subject to availability 2Filmed 3Not with Bernard Magee

2017 FILMING

Bernard Magee, above, will be returning to Denham Grove, Uxbridge, early next year. There, he will deliver his seventh series of six seminars in the course of three bridge filled days.

The audience, drawn as it is, from all over the country will be encouraged by Bernard to fully immerse themselves in the subject matter. He will do this by using his charismatic charm and teaching skills.

The subject of each of the talks is listed in the adjacent advert. At the end of each seminar, Bernard answers questions before going to lunch (or dinner).

The audience can do likewise or go to a table in one of the bridge rooms for a session of supervised play. The focus is on using the new skill taught by Bernard, particular to the lecture just given. For this part of the event you will be in the hands of an experienced team, all on hand to help guests make the most of the occasion, answering and explaining at the bridge table.

I hope that more or less explains how it works, but if you do need to know more, give my office a ring.

Please be aware that the filming takes precedence. You are there to be part of it all and feel the buzz.

All good wishes,

Mr Bridge

Page 8 BRIDGE April 2016

TRAVEL INSURANCEFor your own quote from

( 01268 524344www.covercloud.co.uk

QPLUS 11

Mr Bridge ( 01483 489961www.mrbridge.co.uk

l Help and Hint buttons

l Displays on HD and large screens

l Comprehensive manual

l Feed in your own deals

l Minibridge option

l 5,000 preplayed hands for teams and 4,000 preplayed hands for matchpoint pairs

£99Receive QPlus 11 now and QPlus 12 when ready at the

end of OctoberN

W ES

1. Dealer East. Game All.

♠ A K 9 ♠ J 8 7

♥ K J 3 ♥ A 7 4

♦ Q J 7 6 ♦ A K 4 3

♣ A 10 4 ♣ Q 9 2

West North East South

1NT Pass

?

3NT. You have 18 HCP and your partner has shown 12-14. This gives you a maximum of 32 points, which is not usually enough, with two balanced hands, to make a slam. You do have a ten and nine, but the 4-3-3-3 shape of the hand is the worst you can have. You should settle for game and bid 3NT.

Your partner has the maximum 14 HCP but there are only 9 top tricks.

NW E

S

2. Dealer East. Game All.

♠ A J 4 3 2 ♠ K Q 8 7 5

♥ 3 ♥ A 7 6 4

♦ A 5 2 ♦ 3

♣ Q J 7 6 ♣ A 4 3

West North East South

1♠ Pass

?

4♥. You have great support for spades and the strength for game, so you might respond 4♠. However, 4♠ usually shows a weaker, distributional type of hand with very little slam potential. Your hand will have a good chance for slam if it fits well with the opening hand. The ideal bid is one that shows your shape and allows your partner to judge.

A splinter bid is perfect: it shows SPL – Support (4+ cards) Points (11+ HCP)

and a Lack of cards in the suit you bid: 4♥.

Your partner fits perfectly: he can win his ♥A and take ruffs in your hand, so he would bid Blackwood and eventually settle for 6♠.

NW E

S

3. Dealer East. Game All.

♠ A Q 4 3 ♠ K 2

♥ 4 ♥ K Q 9 8 5 3

♦ Q J 6 3 ♦ K 9 8

♣ A K Q J ♣ 7 6

West North East South

1♥ Pass

?

1♠. You have 19 HCP and should therefore be thinking of a slam already. However, with three different suits to show, you should take your time. As long as you change the suit your partner will have to bid again, and then you can bid a new suit later and keep the auction going. However, remember that if you do not find a fit, you need about 33 points for 6NT which is not certain.

So you respond 1♠ to leave as much room as possible, then over your part-ner’s 2♥ rebid you could bid 3♣ (a new suit at the three-level is forcing). Your partner now bids 3NT and you can fol-low this up with 4NT. This is not Black-wood: it is inviting your partner to slam. He has shown about 12-15 points and you are hoping that he is in the upper range. However, your partner is mini-mum and passes 4NT. The final auction should be something like:

West North East South

1♥ Pass

1♠ Pass 2♥ Pass

3♣ Pass 3NT Pass

4NT All Pass ■

Answers to Bernard Magee’s Bidding Quizzes 1-3

on the Cover

Mr Bridge , Ryden Grange, Knaphill, Surrey GU21 2TH

( 01483 489961 www.mrbridge.co.uk/mrbridge-shop

System Requirements: Windows XP, Vista, 7, 8 or 10, 8mb RAM, CD-ROM

BERNARD MAGEE’S INTERACTIVE TUTORIALS

ACOL BIDDING

l Opening Bids and Responses

l Slams and Strong Openings

l Support for Partnerl Pre-emptingl Overcallsl No-trump

Openings and Responses

l Opener’s and Responder’s Rebids

l Minors and Misfitsl Doublesl Competitive

Auctions

ADVANCED ACOL BIDDING

l Basics

l Advanced Basics

l Weak Twos

l Strong Hands

l Defence to Weak Twos

l Defence to 1NT

l Doubles

l Two-suited Overcalls

l Defences to Other Systems

l Misfits and Distributional Hands

DECLARER PLAY

l Suit Establishment in No-trumps

l Suit Establishment in Suits

l Hold-ups

l Ruffing for Extra Tricks

l Entries in No-trumps

l Delaying Drawing Trumps

l Using the Lead

l Trump Control

l Endplays & Avoidance

l Using the Bidding

ADVANCED DECLARER PLAY

l Making Overtricks in No-trumps

l Making Overtricks in Suit Contracts

l Endplays

l Avoidance

l Wrong Contract

l Simple Squeezes

l Counting the Hand

l Trump Reductions & Coups

l Playing Doubled Contracts

l Safety Plays

DEFENCE

l Lead vs No-trump Contracts

l Lead vs Suit Contracts

l Partner of Leader vs No-trump Contracts

l Partner of Leader vs Suit Contracts

l Count Signals

l Attitude Signals

l Discarding

l Defensive Plan

l Stopping Declarer

l Counting the Hand

£81

£66£96

£76

£76

FIVE-CARD MAJORS &

Strong No-Trump

l Opening Bids & Responses

l No-Trump Openings

l Support for Partner

l Slams & Strong Openings

l Rebids

l Minors & Misfits

l Pre-empting

l Doubles

l Overcalls

l Competitive Auctions

£89

BEGIN BRIDGE ACOL VERSIONl Card Play Technique

l Planning Card Play

l Bidding Balanced Hands

l Bidding Suits

l Responding to a Suit

l Supporting Partner

l Responding to 1NT

l Stayman

l Strong Two Opening and Response

l Overcalls

l Doubles

l Pre-empting

l Defence against No Trump Contracts

l Defence against Suit Contracts

BETTER BRIDGE

l Ruffing for Extra Tricks

l Doubling and Defence Against Doubled Contracts

l Play and Defence of 1NT Contracts

l Finding and Bidding Slams

l Making the Most of High Cards

l Competitive Auctions

£69

£66

MACCompatible

Any 6 for £299

Page 10 BRIDGE April 2016

Standard Faces, with or without bar codes. Unboxed.

6 red/6 blue £19.95 30 red/30 blue only £60

Mr Bridge Premium Quality Cards

Available from The London Bridge Centre. ( 020 7288 1305 www.bridgeshop.com

NW E

S

4. Dealer West. Love All.

♠ A K 2 ♠ 7 6 5

♥ K J 7 6 ♥ A Q 4 3

♦ A 9 4 ♦ K 8 7

♣ A 8 7 ♣ K 3 2

West North East South

1♥ Pass 3♥ Pass

?

3NT. Although you have 19 HCP, slam is not likely because of your shape. In fact, there are 7 losers in the hand using the losing trick count. However, game is certainly on and you might as well describe your hand by rebidding 3NT. Your partner is unlikely to leave you there unless he is extremely flat himself, in which case 3NT might score better than 4♥ because there will be no ruffs.

Here, your partner is 3-4-3-3 as well and might choose to pass. There are like-ly to be ten tricks in both game contracts, but 3NT gives you the extra 10 points.

NW E

S

5. Dealer West. Love All.

♠ K 10 4 ♠ A 2

♥ K Q 7 6 5 ♥ A 9 4 3

♦ 2 ♦ Q 7 6

♣ A K Q 4 ♣ J 7 6 5

West North East South

1♥ Pass 3♥ Pass

?

4NT. You have 17 HCP, five hearts and a singleton diamond: your hand is worth 20 total points (4 losers). Adding this to your partner’s expected 11 gives 31 and with a big fit 30 is your target to contemplate slam.

You have control of every suit and the

number of aces your partner holds will surely give you the chance to place the final contract. If he shows one, you set-tle for 5♥, if he shows two you go for 6♥ and if he shows three, you might contem-plate 7♥ although he is unlikely to have anything else so 6♥ will still probably be high enough.

Here, your partner shows two aces and 6♥ makes in comfort.

NW E

S

6. Dealer West. Love All.

♠ K 8 7 6 5 4 3 ♠ A Q 9 2

♥ Void ♥ A J 8

♦ 7 6 ♦ 9 8 3 2

♣ A K Q J ♣ 7 6

West North East South

1♠ Pass 3♠ Pass

?

4♣. You have quite a hand and, with your part-ner showing support, the strength of your long suit is not such a worry. You have just four losers so there is slam potential, but it all depends where your partner’s strength lies. If he has good diamonds, then slam might make, but if he has strong hearts in-stead, then his strength is wasted and 4♠ is high enough. When you need to be suit-specific, then Blackwood is no good, you need cue bidding. Bidding a new suit after you have agreed to play in a major is not natural, it shows the potential to bid high-er. Above three-of-a-major, it shows slam potential and control in the suit bid – you should bid 4♣. You are hoping that your partner can return the compliment and show where his strength lies. If he bids 4♦, you would be interested in a slam, but if he has no strength in diamonds slam would be no good.

Here, your partner would bid 4♥ and you should sign off in 4♠, expecting to have two losing diamonds. ■

Answers to Bernard Magee’s Bidding Quizzes 4-6

on the Cover

There are Three

Kinds of

Accountant . . .

Those who

can add up and

those who can’t.

Made in the UK from 100% pure cotton © Mr Bridge

SET 27 Leads Bernard takes you through the basic leads and the importance of your lead choice. If you start to think about your partner’s hand, you will get better results. (95 mins.)

8 Losing Trick Count A way of hand evaluation for when you find a fit. Bernard deals with the basics of the LTC then looks at advanced methods to hone your bidding. (92 mins.)

9 Making a Plan as Declarer Bernard explains how to make a plan then expands on how to make the most of your long suits, both in no-trumps and suit contracts. (87 mins.)

10 Responding to 1NT This seminar deals with Transfers and Stayman in detail. The 1NT opening comes up frequently, so having a good, accurate system of responses is paramount. (93 mins.)

11 Signals & Discards This seminar deals with Count, Attitude and Suit-preference signals: aiming to get you working as a partnership in defence. (92 mins.)

12 Endplay Bernard takes you through the basics of endplays before showing some hands where you can take extra tricks, then looks at how to avoid being endplayed. (80 mins.)

SET 11 Ruffing for Extra Tricks This seminar deals with declarer’s use of ruffing to make extra tricks and then looks at how the defenders might counteract this. (74 mins.)

2 Competitive Auctions This seminar focuses on competitive auctions from the perspective of the overcalling side and then from the opening side in the second part. (86 mins.)

3 Making the Most of High Cards This seminar helps declarer to use his high cards more carefully and then looks at how defenders should care for their high cards. (83 mins.)

4 Identifying & Bidding Slams The first half of this seminar identifies when a slam might be on. The second half covers some slam-bidding techniques. (96 mins.)

5 Play & Defence of 1NT Contracts This seminar looks at the most common and yet most feared of contracts: 1NT. The first half looks at declaring 1NT and the second part at defending. (88 mins.)

6 Doubling & Defence against Doubled Contracts The first half of this seminar explores penalty doubles and the second half discusses the defence against doubled contracts. (88 mins.)

BERNARD MAGEETUTORIAL DVDS

SET 313 Hand Evaluation Going beyond just the point-count is important. Reaching & making 3NT on 24 HCP; and avoiding 3NT on 26 HCP when there are only 7 or 8 tricks. (110 mins.)

14 Pre-Emptive Bidding The art of pre-empting is so important in the modern game. Understanding the right hands to bid up on and realising the importance of position and vulnerability. (96 mins.)

15 Splinter & Cue Bids Splinter bids are a vital tool to add to your slam bidding armoury & try your hand at Italian style cue bidding. (116 mins.)

16 Avoidance Play As declarer, an important tactic is to be in control of the defenders: avoiding a particular defender getting the lead. As a defender, you can try to make sure the right player gets the lead. (88 mins.)

17 Play & Defence at Pairs Duplicate Pairs is the game most of us play and getting used to the tactics will make a lot of difference to your performance. (90 mins.)

18 Thinking Defence By far the hardest aspect of bridge, but if you can improve your defence your results will quickly improve. Learn how to think through the defence. (87 mins.)

SET 419 Defensive Plan Looking at your own hand, then at dummy and envisaging how partner’s hand will allow you to make a plan for the defence. (112 mins.)

20 Further Into the Auction The first two bids of an auction are usually easy, but beyond that the complications increase. Learn how to ‘talk’ to your partner during the bidding. (95 mins.)

21 Weak Twos It is important to bid more in the modern game and weak twos are an important choice for the competitive player. (104 mins.)

22 Trump Control Handling the play of the hand when trumps break badly is an important attribute: playing calmly and using a variety of tactics to pave the way to success. (76 mins.)

23 Sacrificing An exciting aspect of the auction is outbidding your opponents and going down, but gaining by doing so. Learn to bid more aggressively. (105 mins.)

24 Improving Bridge Memory Remembering every card is a dream for most of us. However, learn ways in which to remember the important things. (90 mins.)

SET 525 Defence as Partner of the Leader Defence is the hardest aspect of the game, it is where most players can make great progress. (104 mins.)

26 Aggressive Bidding at Duplicate Pairs Years ago, you needed 13 HCP to open the bidding and rarely competed for a partscore. Now its usual to open lighter and compete for every hand. (114 mins.)

27 Strong Opening Bids Managing your strong bids carefully can give you joy, particularly when you have a neat bidding sequence to a lovely slam. (122 mins.)

28 Take-Out Doubles Bernard deals with basic take-out doubles and their responses, then progresses to talk about competing for every partscore. (99 mins.)

29 Suit Establishment in Suit Contracts Long suits are powerful things: Bernard tries to get across his passion for them by showing you how to develop your extra tricks. (81 mins.)

30 Landy / Defending Against a 1NT Opening Competing against a 1NT opening allows you to challenge for the partscore. Bernard talks about competing over 1NT in general and then about Landy. (85 mins.)

Mr Bridge ( 01483 489961 On-line shop:

www.mrbridge.co.uk/shop

£105 SET of 6

£25 per DVD

Buy a Set Get 1 Free

All prices include postage – UK only.

Page 12 BRIDGE April 2016

DUPLICATE BRIDGE RULES

SIMPLIFIED(otherwise known as the Yellow Book)

by John Rumbelow and revised by

David Stevenson

Available from Mr Bridge ( 01483 489961 www.mrbridge.co.uk

only

£595

Mr Bridge Celebration

Party Tie

£15 including postage &

packing

( 01483 489961www.mrbridge.co.uk

NW E

S

7. Dealer East. Love All.

♠ A J 4 3 ♠ K Q 9 8 5

♥ J 7 ♥ A

♦ 6 4 3 ♦ Q J

♣ K Q 10 3 ♣ A J 9 8 7

West North East South

1♠ Pass

3♠ Pass 4♣ Pass

?

4♠. You respond 3♠ to show your 10-12 points (8 losers). Your partner then bids 4♣, which is a slam try: he should have extra strength which together with yours suggest 12 tricks are possible. When your partner cue bids in one suit, he is usually looking for you to show your strength in the other suits. However, your strength lies only in the black suits, so you go back to 4♠.

Your partner cue bid instead of using Blackwood because he needed you to have strength in diamonds (or shortage) to give slam a chance.

NW E

S

8. Dealer West. Love All.

♠ A K J 8 5 ♠ Q 9 7 6 4

♥ J 6 3 ♥ K Q 7 4

♦ 4 ♦ K Q 8

♣ K Q 7 6 ♣ 3

West North East South

1♠ Pass 4♣1 Pass

?1Splinter bid – showing shortage in clubs

and a raise to 4♠

4♠. Your partner should have 11+ HCP, a singleton (or void) in clubs and 4+ cards in spades. How does your hand fit with a singleton club?

An ace is the perfect holding or even four low cards because your low cards can be ruffed away. However, other high

cards are a bit of a waste. Although your king-queen will eventually make one trick, it is rather a waste that one hand has a singleton and the other 5 HCP and yet you are still losing one trick in the suit.

If you fit well with a singleton you can sometimes make slams on 24 or 25 com-bined points, but without a good fit you still need the usual 30 points.

The negative response to a splinter is to return to your suit: 4♠. This says part-ner, ‘I do not have interest in slam be-cause I do not fit well with you.’

NW E

S

9. Dealer West. Love All.

♠ A K J 8 5 ♠ Q 9 7 4 3

♥ Q 2 ♥ A K 5 3

♦ A 6 ♦ K 9 5

♣ 9 8 7 6 ♣ 2

West North East South

1♠ Pass 4♣1 Pass

?1Splinter bid – showing shortage in clubs

and a raise to 4♠

4♦. Your partner should have 11+ HCP, a singleton (or void) in clubs and 4+ cards in spades. How does your hand fit with a singleton club?

Four low cards is a great holding – your partner should be able to ruff your losers away. Furthermore, your high cards will be together with his high cards and there may well be the chance of a slam.

To show your excitement, you can cue bid or use Blackwood depending on the type of hand you hold. Here, with weak-ness in hearts you should cue bid a suit where you hold strength to see if your partner has something in hearts. You bid 4♦ and now your partner can take control: he uses Blackwood and goes for 6♠ when you show three key cards (in-cluding the king of trumps). Just 26 HCP between you, but with the perfect fit your slam should be straightforward. ■

Answers to Bernard Magee’s Bidding Quizzes 7-9

on the Cover

CLUB INSURANCE

Every club should be covered and my inclusive package, to suit clubs of up to 100 members for less than £75 per year, is the right package at the right price.

Ring FIDENTIA for a quote

( 020 3150 0080

BRIDGE April 2016 Page 13

This DVD deals with this often difficult declarer play tech-nique. Very often when you are

declaring you prefer one defender to have the lead rather than the other. Labelling the two defenders as the goodie and the baddie, your aim is to avoid the baddie getting the lead.

The baddie will often have winners in his hand, or will be able to make a killing switch. Look at his hand and try to identify the goodie and baddie:

NW E

S

♠ K 8 6 5 4

♥ K Q J 10

♦ 7 3

♣ 4 3

♠ A Q J 3 2

♥ 8 7 5

♦ K 2

♣ A 7 6

4♠ by South. Lead: ♣Q.Identifying your weakness is your first job and that is in diamonds, but you should only be worried if East gets the lead because if West chooses to lead diamonds your king will make a trick. East is your baddie, so you need to keep him off lead.

The opening lead is the ♣Q which is ducked to you, what is your plan?

You do not know who has the ♥A, so if East has it you cannot keep him off lead, but if West has it, then you have a chance because the defenders have allowed you to choose who wins their club trick. As so often you have to be thinking at trick 1: ducking West’s lead leaves him to play to trick 2 and he cannot lead a diamond without giv-ing you your ♦K, so he continues with another club. You draw trumps and knock out the ♥A and all West can do is take his ♦A. Had you won the first trick and played the same way, then

West leads a club for his partner to win and a diamond switch will defeat you.

NW E

S

♠ K 8 6 5 4

♥ K Q J 10

♦ 7 3

♣ 4 3

♠ 10 9 ♠ 7

♥ A 3 2 ♥ 9 6 4

♦ A 9 8 6 ♦ Q J 10 5 4

♣ Q J 10 2 ♣ K 9 8 5

♠ A Q J 3 2

♥ 8 7 5

♦ K 2

♣ A 7 6

Many of the techniques in the DVD are difficult, but then in the second half I talk about the ideas from the perspective of defenders, which is even harder. An expert East might manage to play the ♣K on trick one, forcing declarer to win the trick and East might then discard the ♣9 on the second round of trumps to suggest he held the ♣8 allowing West to win the ♥A, lead the ♣2 to East’s eight and fi-nally a diamond switch would defeat the contract. I do not go as deep as all that in the DVD, but try to talk about some ideas that are within the grasp of club players: ducking, overtaking, unblocking and switching are some of the ideas, as well as a few signals too.

Here is a classic kind of deal which exhibits an important part of defen-sive play (shown in next column).

West leads the ♣K, which wins the first trick, and then the ♣Q which again is allowed to win. This is the key point of the play: West knows that his club suit is redundant because he has no entry. If East had the ♣A he would have already played it. A lot of defend-ers would continue with another club, but that is pointless. Declarer wins, knocks out the ♥A and claims nine tricks: four hearts, three spades and the two minor suit aces.

NW E

S

♠ A K 6 5 4

♥ J 10

♦ J 7 4 3

♣ 4 3

♠ 10 9 3 2 ♠ J 7

♥ 3 2 ♥ A 9 6 4

♦ 9 8 ♦ K Q 10 5

♣ K Q J 10 2 ♣ 9 8 5

♠ Q 8

♥ K Q 8 7 5

♦ A 6 2

♣ A 7 6

West North East South

1♥

Pass 1♠ Pass 1NT

Pass 3♦ Pass 3NT

All Pass

Instead West remembers that he is in a partnership and just because his suit is redundant it does not end the defence. He aims to find his partner’s strength and establish some tricks in his hand. Spades are out of the ques-tion and leading in to South’s heart suit is unlikely to be of much use, so West chooses the ♦9. This quickly develops two tricks for your partner, which together with the first two clubs and the ♥A defeats 3NT.

By ducking the first two tricks de-clarer is aiming to isolate the clubs in one hand. As you saw this would have worked if West continued clubs. Since the defenders can see their own cards, they can counteract declarer’s avoidance play. Had West held the ♥A, he would have continued with clubs, knowing that he will get the lead later.

This DVD aims to widen the hori-zons of declarers – showing the power that declarer holds over the defenders and how you can often choose which defender gets the lead and thereby make sure that the fate of the contract is in your control. At the same time of course, defenders can be equally can-ny and try to outwit the declarer. ■

AvoidanceBernard Magee DVDs – Number Sixteen

Page 14 BRIDGE April 2016

Answers to Bernard Magee’s Bidding Quizzes 10-12

on the Cover

NW E

S

10. Dealer East. Love All.

♠ K Q 4 ♠ A J 10 7 6 5 3

♥ K Q 5 ♥ 4 3

♦ K Q J 4 ♦ 9 8

♣ K Q 5 ♣ 8 6

West North East South

3♠ Pass

?

4♠. This hand is relatively straightforward: al-though you do have 21 HCP, there is no chance that your partner can have more than two aces and, therefore, a slam has no chance.

You might contemplate 3NT, but you may well be exposed in one suit and with aces to be forced out, your game might be in danger. 4♠ is the simple bid – going for the game you know you can make.

NW E

S

11. Dealer East. Love All.

♠ K 9 8 ♠ A J 10 7 6 5 3

♥ 2 ♥ 4 3

♦ A K 7 6 5 4 2 ♦ 9 8

♣ A 7 ♣ 8 6

West North East South

3♠ Pass

?

4NT. Just 14 HCP so you might think the question is about whether you should bid game or not. However, the potential of your hand is huge: a ten-card spade fit and surely the chance to establish six tricks in diamonds. It looks as if there will be twelve tricks available as long as your partner has an ace. It has to be worth bidding Blackwood and then bidding slam if your partner shows one ace.

On a club lead, you win the ace, draw trumps in two rounds and then play three

rounds of diamonds ruffing the third. As long as trumps break 2-1 and diamonds no worse than 3-1, you make all thirteen tricks. (Of course, you make only twelve if they lead hearts).

NW E

S

12. Dealer West. Love All.

♠ J 3 ♠ A K Q 4

♥ K Q 7 6 5 ♥ J 2

♦ K Q 4 3 ♦ A J 2

♣ A 2 ♣ K 6 5 4

West North East South

1♥ Pass 1♠ Pass

2♦ Pass 3♣1 Pass

3NT Pass 4NT Pass

? 1Fourth suit forcing

6NT. You open 1♥ and then rebid 2♦, showing your 5-4 shape, but then your partner bids the fourth suit: 3♣. This does not promise clubs, but is simply a forcing bid, requiring you to bid again. With a stopper in clubs you bid 3NT. Now your partner bids 4NT and you have to work out what he wants.

A 4NT bid immediately after another no-trump bid is not ace-asking, it is a quantitative bid inviting slam. It asks you to look at your hand and judge whether you are minimum or maximum. If you are minimum then pass, if maximum raise to slam.

You have shown a minimum opener so your hand should be in the range of about 12-15. You are certainly top of your range and your ♠J in partner’s suit might be of some use. You should bid 6NT.

You can see your partner’s problem – he has 18 points balanced but he knows he needs about 33 combined points to make 6NT, so he keeps making forcing bids until he is in a position to invite you to slam. Had you held just 12 points then you would have finished in 4NT. Instead, you finish in the excellent slam. ■

Life’s a GAME

but

BRIDGE

is

SERIOUSMade in the UK

from 100% pure cotton © Mr Bridge

www.brianbridge.netApp-based scoring for duplicate bridge

Individual – Pairs – TeamsCafé Bridge

l Players enter scores on their iPad/iPhone/Android

l No startup costl Play, direct or watch from

anywhere in the world

Step into the future of bridge scoring

[email protected]

BRIDGE April 2016 Page 15

What call do you make with the North cards on the hand below after partner

has opened 1♥?

NW E

S

♠ A 2

♥ K 10 6 3

♦ A J 7

♣ A J 5 4

Playing Jacoby, bid 2NT. Jacoby is used in response to partner’s 1♥/1♠ open-ing bid. It shows four-card support for partner’s major and values for game.

Responses to JacobyThere are many different variations of continuations to Jacoby, however, the following is a simple and effective, and a commonly used variation. Assume you have opened 1♥ and partner has responded 2NT, Jacoby:

l 3♣/3♦/3♠ shows a second suit, at least five hearts and four of the suit bid.

l 3♥ shows a hand that is better than minimum, but with no shortage or a second suit.

l 3NT shows a balanced hand of 15+ points and is forcing.

l 4♣/4♦/4♠ are splinters.l 4♥ is your weakest action, saying

that you have no interest in a slam.l 4NT, of course, you can go straight

to Blackwood if you have the right sort of hand.

Returning to the hand above, your partner rebids 3NT, showing 15+ bal-anced. You now know that you have enough points for a slam, so bid 4NT

which, despite being a raise of a no-trump bid, is RKCB for hearts as this is already the agreed trump suit. Part-ner responds 5♦ showing one key card. You are out of room to ask about the queen, but, holding the requisite values you can take a gamble and try 6♥.

West leads the ♦10. How do you plan the play?

NW E

S

♠ A 2

♥ K 10 6 3

♦ A J 7

♣ A J 5 4

♠ K 10 8 4

♥ A Q J 9

♦ K Q 5

♣ 10 3

Your best line is to try to ruff two spades in dummy. Since all your trumps are high in hand, you can ruff them with the king and ten. Don’t be tempted to draw any trumps first, since you will need them to cross safely back to hand. Provided two rounds of spades stand up you are virtually home. Ruff the third one high, cross back to a trump, ruff the fourth one high and cross back with a trump to draw trumps. Unless someone holds five spades, four trumps (requiring you to play four rounds) and the ♣A, you are now home.

Advantages of JacobyWhat are the advantages of using Jacoby instead of bidding four of partner’s major?l It leaves more room to explore

for slam if either player has extra values, especially for cue bidding

l It frees up a bid of 4 of the major to be used pre-emptively.

What call do you make with the North cards on the hand below after partner has opened 1♥?

NW E

S

♠ J 10 9 6 2

♥ Q J 10 3 2

♦ Void

♣ 9 8 7

Raise directly to 4♥. Partner won’t expect you to have a good hand since otherwise you would have used Jacoby. Instead you can put pressure on the opponents by making a pre-emptive raise. This was the full deal:

NW E

S

♠ J 10 9 6 2

♥ Q J 10 3 2

♥ Void

♣ 9 8 7

♠ A 5 4 3 ♠ 7

♥ Void ♥ 9 6 5

♦ A 9 5 4 3 ♦ K Q J 7

♣ Q 10 5 4 ♣ A K 6 3 2

♠ K Q 8

♥ A K 8 7 4

♦ 10 8 6 2

♣ J

You can see from the whole deal that, barring the miracle defence of ♠A lead, and a spade ruff followed by an underlead of a club to West for another ruff, your contract will make.

Heather’s Private Bridge Lessons

Jacoby 2NT

Page 16 BRIDGE April 2016

Now look at what the opponents can make. A small slam is easy, and West can even make 7♣, but is it obvious for either of them to bid?

What do you call with the South cards on the hand below after partner has responded 2NT to your 1♥?

NW E

S

♠ 3 2

♥ A Q 10 9 4

♦ K Q J 6

♣ K 4

Bid 3♦ showing a second suit and not a complete minimum. Partner now cue bids 3♠. Knowing that partner has a spade control, you can proceed safely to RKCB, over which partner shows two key cards and you bid 6♥.

NW E

S

♠ A Q 6 5

♥ K J 8 5

♦ A 2

♣ Q 6 5

♠ 3 2

♥ A Q 10 9 4

♦ K Q J 6

♣ K 4

West leads the ♦3. You win and draw trumps in two rounds. How do you plan the play?

If you can guess who holds the ♣A, you will be home by leading a low club from the hand to the right of that defender. The defender cannot afford to rise with the ace since this will allow you to score both club honours and discard a spade from hand on the ♣Q, yet if they duck, you can discard dummy’s remaining two clubs on the diamonds. You may decide to play East for the ace since there is a chance that West would have led it, if held. Even if you misguess this, there is still the possibility of the spade finesse.

What do you call with the South cards on the hand below after partner has responded 2NT to your 1♥?

NW E

S

♠ A 7 6

♥ K Q 7 4 2

♦ A 4 2

♣ K 9

Rebid 3♥, showing a better than minimum hand without any other convenient call to make, so, for example, you would be denying a second suit. When partner co-operates with a cue bid you can investigate aces and kings, but will stop in 6♥ when you discover a king is missing.

NW E

S

♠ K 5

♥ A 10 5 3

♦ Q J 10 5

♣ A J 4

♠ A 7 6

♥ K Q 7 4 2

♦ A 4 2

♣ K 9

West leads the ♠3. How do you plan the play?

The contract looks pretty secure, but you should be careful to guard against a potential bad trump break. Begin with a trump to the king. When East fails to follow you can still afford to draw four rounds of trumps, taking the marked finesse, since your third spade will disappear on the fourth round of diamonds.

What do you call with the South cards on the hand below after partner has responded 2NT to your 1♥?

NW E

S

♠ A K 6

♥ A 9 6 3

♦ J 10 9

♣ A 9 3

Jacoby 2NT continued

Daily bridge on board, bridge fees included. Mr Bridge welcome &

farewell drinks parties. Partners for single players. No single supplements

on selected twin grades of cabin.

Terms and conditions apply – see Fred. Olsen 2016/17 worldwide brochure

For reservations call Mr Bridge

on 01483 489961www.mrbridge.co.uk

• Southampton • Funchal, Madeira • Santa Cruz, La Palma • Santa Cruz, Tenerife • Las Palmas, Gran Canaria

• Arrecife, Lanzarote • Lisbon, Portugal • Southampton

Sails from Southampton7th January 2017 • 13 nights

Balmoral • L1701

Balmoral

Great value Mr Bridge fares:

Inside twin cabins from £949pp

Outside twin cabins from £1,139pp

Balcony cabins from £1,709pp

Suites from £1,949pp

Twin outside cabin for sole occupancy

£1,199pp

Canaries & Madeira Winter Warmth

FOC311297_Mr_Bridge_Adverts_261.5x57mm_W/C_27.07.15_FV.indd 329/07/2015 14:42

BRIDGE April 2016 Page 17

Rebid 3NT, showing a strong no-trump, 15+ points. Partner now proceeds with Roman Key Card Blackwood and settles for 6♥ after discovering that you don’t hold the ♥Q.

NW E

S

♠ Q J 3

♥ K J 4 2

♦ A K Q 8

♣ K 7

♠ A K 6

♥ A 9 6 3

♦ J 10 9

♣ A 9 3

West leads the ♠10. How do you plan the play?

Provided you don’t lose two trump tricks your contract is secure, therefore playing teams or rubber bridge you should make a safety play. Begin by cashing the ♥K and then lead toward the ♥A-9, inserting the nine if East follows low. If East fails to follow, you can rise with the ace and lead towards the jack. In this way you can guard against any 4-1 break.

Playing duplicate pairs, things are a little more complicated. If your contract is a normal one, you should reject the safety play since it may well lead to you giving up on the overtrick that the straightforward finesse will give you.

However, here there are so many points around that some pairs will certainly reach 6NT, and others may get to 7♥, in which case the safety play is the best way to go for a good score since you need to hope that this is necessary. You may need to consider the quality of the other pairs.

What do you call with the South cards on the hand below (see next column) after partner has responded 2NT to your 1♠?

Jump to 4♠. You scarcely have an opening bid at all, and with no aces you need to warn partner off.

N

W ES

♠ K Q 10 7 3

♥ Q J 10

♦ 6 5 4

♣ K 6

West leads the ♥4 to East’s ♥A. East follows with the ♥K and then plays a third heart.

You win in hand and draw trumps in three rounds. How do you plan the play?

NW E

S

♠ A 9 4 2

♥ 5 3 2

♦ A Q 9 8

♣ A 7

♠ K Q 10 7 3

♥ Q J 10

♦ 6 5 4

♣ K 6

You need to avoid two diamond losers in your spade game, which you can guarantee by means of an elimination. Cash the clubs ending in hand and lead a diamond, inserting the nine if West follows low. East may win with the ten or jack, but is now endplayed. If West puts up the jack or ten, you can counter this by playing the queen. Then if East wins the king, he is endplayed.

If the Jacoby bidder jumps to game over your second suit, that is also showing a minimum hand. When you are in a game-forcing sequence, jumps to game are often weaker than bidding three of the suit. This is known as the principle of fast arrival. The advantage is that when you have extra values, you can show partner your strength whilst still remaining at a low enough level to start cue-bidding.

So, after 1♠-2NT, 4♠ is weaker than 3♠.After 1♠-2NT-3♦, 4♠ is weaker than 3♠.After 1♠, 4♠ is weaker than 2NT. ■

Jacoby 2NT continued

• Fly to Malaga (Granada)Cruise to • Cadiz (Jerez) • Funchal • La Palma • Arrecife • El Marsa (Western Sahara Desert) • Agadir

• Marrakesh (overnight hotel*) • Casablanca(Rabat) overnight onboard • Tangier

• Malaga • Fly home

Join us on a magnificent journey from Andalusian Spain to Atlantic islands and the unique landscapes

of the Western Sahara Desert

Great value Mr Bridge fares

Standard Inside from £1,850pp

Standard Outside from £2,450pp

Premium Outside from £2,650pp

Single supplement just 10%

November 6, 2016 - 15 days from Malaga to Malaga

www.mrbridge.co.uk

For reservations call

on 01483 489961

Terms and conditions apply – see brochure or website for details* No bridge on overnight hotel stay

REVISED ITINERARY

Madeira,Canaries

& Moroccohosted by

WILL and SYLVIA PARSONS with Judy and Martin Holcombe

2101 Mr Bridge third page ad 161106BR _Layout 1 2

Page 18 BRIDGE April 2016

Catching Up with Sally Brock

It has been quite a busy time from both a bridge and a non-bridge point of view. We played in the

National Point-a-Board teams in the middle of January. I played with Chris Dixon, and Barry played with Robert Sheehan. We did terribly. I had an enjoyable weekend and Chris was a charming partner but the bridge was truly awful. We were nowhere near qualifying for the final, and even in the consolation Swiss teams we were nowhere near the top.

This was a freak hand from the qualifying stages:

NW E

S

Dealer South. Love All.

♠ Void

♥ A Q J 9 7 5 4 2

♦ A 10 9 7 5

♣ Void

♠ A K ♠ 10 5 3

♥ K 10 3 ♥ 8

♦ Q 2 ♦ 8 6 4 3

♣ Q J 10 8 6 5 ♣ K 9 4 3 2

♠ Q J 9 8 7 6 4 2

♥ 6

♦ K J

♣ A 7

I’m sure Chris didn’t like putting that North hand down as dummy, but after I opened 4♠ the choice was between passing and bidding 6♥. On a club lead against 4♠, I did not want to risk any finesses as the ten of spades dropping doubleton was a reasonable chance. So I won the king of clubs and played the ace and queen of hearts. For some reason East ruffed in with the ten of spades and now I was home with no more guessing. Actually, I should have made an overtrick because it was unlikely to cost to play the king and jack of diamonds, hoping that the queen might fall. When it does, I can play a third round pitching my club loser as West has to ruff with his trump trick.

Those Norths who played with less

exuberant partners got the chance to bid their hearts, often at a high level. How should you play 6♥ after a club lead? You could take a heart finesse and then guess diamonds, but it feels better to me to play to ruff a diamond in the dummy. When the queen of diamonds drops, all your problems are over.

The weekend after that was the Lady Milne Trials in which I played with Susanna. We were leading for quite a lot of the event, but in the end made a couple of extremely costly decisions which meant we finished fifth – close but not close enough. I liked the way we bid this hand:

NW E

S

Dealer East. Love All.

♠ 9

♥ A J

♦ A K J 2

♣ A K J 8 5 3

♠ A 10 6 3 ♠ J 8 7 4 2

♥ K Q 3 2 ♥ 10 8 7 6 5

♦ Q 8 7 5 4 ♦ 6

♣ Void ♣ 7 6

♠ K Q 5

♥ 9 4

♦ 10 9 3

♣ Q 10 9 4 2

West North East South

Pass Pass

1♦ Dbl Pass 2♣

Pass 2♦ Pass 2♠

Pass 3♦ Pass 4♦

Pass 6♣ All Pass

Susanna had a difficult bid to find after the 1♦ opening. I think I might have tried 3NT, not wanting to double with so few cards in the majors. As it happened, the double worked well when I made the rather surprising response of 2♣. In response to her second cue bid, my raise to 4♦ was intended to suggest a hand that wanted to bid game but didn’t know which one (I thought she might have

had a very strong hand with, maybe, 4-4-2-3 distribution – and I wanted to encourage her to try a chunky four-card major so we could perhaps play in 4♠). I guess it was a bit lucky that I held the nine of diamonds, but you need a bit of luck from time to time.

Other than bridge I have been busy with my parents who have decided to downsize and move into a retirement village. I think this is an excellent idea as my mother now needs rather more care than my father can provide. So it is a time for sorting out their new home and helping them decide what to do with all the possessions they will not have room for. An ongoing project.

I have recently been appointed head of the young women’s training squad. For a long time there have been three squads: Under 25, Under 20 and Under 15, with a smattering of women/girls in each. However, it was felt that it would be good to give the young women some single-sex training. We have been lucky enough to have had an extremely successful women’s team for a number of years, and it would be a pity if that was to end when the current team retires. So we are going to have squad meetings probably three times a year, as well as some online training, newsletters, etc. If any of you know any young women who might be interested in joining us (enthusiasm is the most important criterion), please let me know ([email protected]).

I am still helping Briony with her various projects. Her charity is the beneficiary of the Charity Challenge Cup this year, so we have been trying to persuade more clubs to enter. The date this year is March 17th but that might have passed by the time you read this. Then there is The Big Bridge Quiz which is now available (closing date April 30th) with a first prize of £1,000. In addition to that, we are organising a Pro-Am type of event on May 6th: Some Enchanted Evening at the Livery Halls near London Bridge. ■

BRIDGE April 2016 Page 19

Possibly the most exciting thing that can happen in our game is to bid and make a grand slam,

especially with a very low point-count. Andy Elliot asks for advice with this deal from a club duplicate.

NW E

S

Dealer North. Game All.

♠ A K Q 10 6

♥ A K 7 2

♦ A 5 4 2

♣ Void

♠ J 8 4 3 ♠ 7 5

♥ Q 9 3 ♥ J 10 4

♦ 9 ♦ 8 6

♣ A 9 7 4 2 ♣ K Q 10 8 6 5

♠ 9 2

♥ 8 6 5

♦ K Q J 10 7 3

♣ J 3

West North East South

2♣ Pass 2♦

Pass 2♠ Pass 3♦

Pass ?

At this stage, Andy bid 5♣, intending it as Exclusion Blackwood, but his partner misunderstood and just rebid 5♦, raised to 6♦ by North.

Even if 5♣ is not Exclusion Blackwood (and many club players do not play the convention), it should show a void when it is at such a high level. The South hand is tremendous, facing a void, and he should certainly jump to 6♦, after which a raise to the grand slam by North would not be

unreasonable.If North is uncertain that his partner

will understand 5♣ as showing a void, the auction may work better if North rebids 3♥. After that South, with a semi-solid suit, should jump to 5♦, and North can raise to the grand slam.

♣♦♥♠

Just to prove that it is not necessary to take masses of rounds of bidding and then use Blackwood, or make a lot of cue bids in order to bid to a good slam, this deal was sent in by Jim Dapre.

NW E

S

Dealer East. E/W Game.

♠ 6 5 ♠ A Q 7 3 2

♥ Void ♥ 8 4

♦ A Q J 10 7 3 2 ♦ K 9 5

♣ A K 5 4 ♣ Q J 7

East opened 1♠ and South overcalled 2♥. Jim asks what he should bid. He saw deeply into the hand. Realising that if he just bid 3♦, North was likely to raise hearts to a high level and then even if East-West did bid the slam, North-South might sacrifice, Jim jumped straight to 6♦.

‘What do you understand by that bid?’ asked North.

‘Sounds like a dip in the lucky barrel!’ replied East.

However, 6♦ was passed out and made in comfort while teammates found the cheap 6♥ sacrifice in the other room. ■

A Good Grand Slam

Sally’s Slam of the Month

Send your slam hands to [email protected]

Fly to Athens to join Aegean Odyssey Cruise to • Gythion (Sparta & Mystras)

• Katakolon (Olympia) • Ithaca • Corfu • Sarande (Butrint) • Kotor Bay

• Dubrovnik (overnight onboard) • Split • Zadar • Ravenna • Venice

• Fly home

The classics of Greece and thebeauty of the Adriatic combine on this fascinating bridge cruise

from Athens to Venice

Great value Mr Bridge fares

Standard Inside from £1,895pp

Standard Outside from £2,350pp

Premium Outside from £2,475pp

Single supplement just 10%

September 27, 2016 - 10 days from Athens to Venice

www.mrbridge.co.uk

For reservations call

on 01483 489961

Terms and conditions apply – see brochure or website for details

Greece& the

Adriatic Seahosted by

BERNARD MAGEEand team

2101 Mr Bridge third page ad 160927BR _Layout 1 2

Page 20 BRIDGE April 2016

QMy partner and I play a fairly simple defence

to 1NT. With 16+ points we always double. Our two-level overcalls deal with situations where we have fewer points, but seven losers or fewer (not vulnerable) and six losers or fewer (vulnerable). A suit overcall at the two-level guarantees 4+ cards in the suit bid; it also promises either that the suit is in fact 6+ cards or that the hand is two-suited, with a distribution of at least 4-5. Every two-level overcall can be passed. Therefore, we believe our two-level overcalls are natural. But should we be alerting them?

Recently I had the following hand in a club bridge session in which no-one had a system card:

♠ 6

♥ 9 6 5

♦ A K Q J 5

♣ Q 10 7 6

After a 1NT opening by RHO, I overcalled 2♣,

which my partner knew meant either 6+ clubs, or clubs and another suit with shorter clubs. She did not alert it. My LHO bid 3♣, asking for a stopper in clubs for 3NT, which his partner had. I promptly led out all my diamonds against 3NT. 3NT was the contract at two other tables, and it went off by two both times.

LHO called the director, who ruled that there was no need to alert the 2♣ bid as it promised 4+ clubs. I notice that page 26 of the 2008 edition of Duplicate Bridge Rules Simplified also says that a non-forcing overcall where the suit may contain only four cards should not be alerted.Gavin Wilson by email.

A This defence, sometimes called Middlesex, is not

really what is meant by a natural bid. There is a presumption with natural bidding that you are bidding your longest suit. So you are required to alert your two-level overcalls. While many pairs in clubs do not have system cards, it is normal to have a system

card if playing anything very unusual. I believe this defence qualifies as very unusual and recommend you to carry a system card, even if no-one else does.

♣♦♥♠

QAfter an opening 1NT (12-14) on my left, my

partner overcalled 2♣. I forgot it was Asptro (showing 5-4 or 4-5 in hearts and another suit) so didn’t alert it. I bid 3♣ to show good club support, but in fact meaningless in our Asptro system. My partner bid 3♥ and at that point I realised my mistake. Neither of us gave anything away behaviourally.

What should I do now? Tell the opponents then and there what the original 2♣ meant? Stay schtum until the end of the auction and explain before the opening lead? Or should my partner do that? Can I now treat the 3♥ (which if we were playing naturally would have been a 3NT try) as a rescue and pass it?Name and address supplied.

A You are required by the laws to summon the director and

tell the opponents in his presence that you have misinformed them by not alerting the 2♣ bid. If asked you explain that it is Asptro.

You are not constrained in any way during the bidding, since you have remembered of your own accord and you can now try to retrieve the situation as best you can.

The same is not true of your partner. He has unauthorised information because after his 2♣ bid you did not alert, so he must take every care not to gain from this. That is why the director should be called: he will warn your partner suitably, and if he feels afterwards your partner has taken any advantage he may adjust.

♣♦♥♠

QSome friends whilst abroad, came across a

multi, weak, but only 4-4 in the majors.

Is this legal in England? They thought it was called Norwegian or Swedish Multi.Alex Mathers by email.

David Stevenson answers your questions on Laws and Ethics

Is Our Defence to 1NT Alertable?

BRIDGE April 2016 Page 21

A It is not a Multi, but it is certainly legal. Some people

call it the Norwegian 2♦, but the more common name is Ekrens 2♦ after the inventor. The Ekrens 2♣ is also legal, opening 2♣ with both majors, weak.

♣♦♥♠

QMy opponent at the club apologised as

his partner had failed to alert his double in the following sequence: 1NT-2♦-Dbl. The 2♦ bid was by my partner and was natural. I said that it was not alertable since I assumed that it was for penalty and we had a friendly discussion about it.

My opinion is based on the memory that Bernard Magee teaches that all doubles are penalty following a no-trump bid or pre-emptive bid by our side and would therefore be the normal meaning and not alertable with only one exception ie 1♦-Pass-1NT-Dbl which would be for take-out and be alertable.

Looking at various EBU books (white, blue and orange), I can see that unless doubles of suit bids are for take-out and doubles of no-trump are for penalty then the alternative meaning must be alerted. It does not as far as I can see then continue to explain what happens after a no-trump bid is overcalled by a suit

and then doubled. I would be interested in your opinion.Crombie McNeil, Faversham.

A You have read the correct rule, but seem not to have believed

it! When your partner doubles a natural suit bid at the one, two or three level, you alert it unless it is for take-out. You say that in that situation Bernard teaches you to play penalty doubles: of course he does, since this, as you put it, is the normal meaning for double, but that does not alter the fact that it is alertable. The alerting of doubles has nothing to do with the normal meaning.

Why not? Well, the EBU tried for years to get people to alert doubles sensibly based on common expectations and failed dismally. The alerting of doubles was terrible, with even good players continually getting it wrong, so they decided to go for the simplest rule possible. The only trouble is that, despite the rule being in place for eight years or so, players (including good ones) do not believe it, and still alert wrongly.

So please remember that it does not matter what you expect of a double, if it is of a natural suit bid below 3NT it is alertable if it is not for take-out.

♣♦♥♠

QPlaying in 4♠, I needed to find the queen

to make my contract, l led the king from hand followed by a small spade towards the ♠A-J in dummy, intending to try the finesse. Sadly my partner played the ace before I could say anything and almost

instantly my RHO followed suit. I accepted the situation and later LHO cashed the queen and my contract was one down. Could I have asked for the jack to be played instead of the ace at any time?Dick Symonds, Market Bosworth, Leicestershire.

A The Law says that if dummy plays a card that declarer

did not name then it may be corrected if attention is drawn to it before each side has played to the next trick. So you can change the card to the jack, and tell dummy off severely!

Dummy has been participating in the play and suggested a play. It has not mattered since your RHO followed suit, but if he had discarded, then the director might adjust to give the defence a trick because of dummy participating.

♣♦♥♠

QWho can bid what after a system error?

West North East South

1NT1 Pass 2♦ 2♥

Pass 112-14

At this point, North asked if 2♦ was natural, West said she should have announced 2♦ as a transfer but forgot. The director was called and said that South could withdraw the 2♥ bid which she did and passed. West then bid 2♥ raised by East to 4♥.

Surely E/W have what amounts to unauthorised information and should either be not permitted to bid, or not permitted to

Ask David continued bid hearts? At club level, many players forget their system agreements, but surely should suffer in some way for doing so?Peter Courtney, La Viñuela BC, Spain.

A Sometimes players get away with such errors because

nothing illegal has occurred. West has no unauthorised information because the question from opponents reminded him and any information from opponents is authorised. East has unauthorised information that something is wrong because of the lack of an announcement, but it does not seem to matter since West has remembered before East does anything.

The 2♥ bid by South is unauthorised to East and West (but authorised to North) but it is not clear how it has affected anything. East bid 4♥ anyway, so unless East might have bid 6♥ otherwise, or unless it affected the play of the hand there does not seem to be any damage.

♣♦♥♠

QThe previous alerting rules stated that bids

over 3NT did not need alerting except for artificial opening bids. The current version is that, ‘An artificial suit bid on the first round is alertable (the first round for this purpose being the opening bid and three subsequent calls).’

In the sequence 1♥- Pass-4♣, with the 4♣ bid being Gerber, does this bid qualify as an ‘artificial suit bid’ and therefore is alertable?John Dawe by email.

Page 22 BRIDGE April 2016

A Yes, 4♣ does not show clubs, so it is artificial

and requires an alert.

♣♦♥♠

QPlaying a weak no-trump (12-14), can one

bid 1NT with a flat hand and 16 points?R Bloodworth by email.

A Yes, you can. If you decide on any particular hand to

deviate somewhat from your system, and your partner does not know, that is fine. If you do it regularly then it becomes part of your system, and you should alter your announcement accordingly.

♣♦♥♠

QCan you please inform me if it is legal to pre-empt

as an opening bid with 13 points in your hand?Connie Pearce by email.

A The simple answer is yes, with reservations. If you

normally have a normal eight points or so, and on one occasion you look at 13 points, perhaps with a singleton king or something, and decide to open 3♥, then that is fine. In fact expert advice is to vary your pre-empts third-in-hand, so in that position, a pre-empt may easily have 13 points.

However, suppose you and your partner decide that whenever you have a seven-card suit you will open 3♥, even with 13 or 14 points, what then? Well, it is still perfectly legal, since

you can play natural three-level opening with whatever strength you like, but you must let the opponents know this. On your system card you should write something, perhaps a range 8 to 14, or a little note, ‘may be stronger than usual.’ I think you should alert it as well, since it will not be what opponents expect.

♣♦♥♠

QWhen the bidding has ended, can the

player on lead ask a question before he plays his face-down card? If he plays a face-down lead and his partner asks a question, can he change his lead as a result of the answer given to his partner?Graham Lavender by email.

A Yes, he may ask a question, in fact this is the normal time

for the defender on lead to ask a question. Every player except dummy may always ask a question whenever it is his turn to call or play. The only time that he may change the lead is if the answer to a question makes it clear he has been misinformed, and only if the director allows the change.

Perhaps two examples will make this clear. After an alerted response of 4♣ to a 1♥ opening, a player is on lead to 6♥. He assumes without asking that the 4♣ is a splinter showing a shortage so decides to lead a trump. His partner asks about 4♣ and is told it is Gerber: now he realises a trump is wrong and a club is better. May he change it? No, he has not been misinformed, it is his own fault he did not ask.

Let me change it slightly, so

Ask David continued that the initial response was 3♣ which was not alerted. As before, the opening leader does not ask, but leads a diamond against 6♥ away from the king because he is worried the diamonds will go away on dummy’s clubs. When his partner asks, he discovers that 3♣ is a mini-splinter, showing a shortage, so there is no reason for a dangerous diamond lead. May he change it? Yes, the failure to alert is misinformation, which has come to light because of partner’s question, and the director will allow a change.

♣♦♥♠

QTwice within the last week a revoke has

occurred. A defender ruffed a trick and immediately played a card of the suit that she had just ruffed. Each time, the directors (two different clubs, different directors of good standing) gave a ruling to withdraw the newly played card, withdraw the trump from the previous trick, let declarer have that trick, which she would have won without the revoke and to continue as if there had been no revoke (apart from the fact that the trump became a major penalty card). Have the Laws changed on this? Looking at the 2007 Laws, the revoke is established in these circumstances. In one instance, it was dummy who noticed it but wasn’t sure if he was allowed to draw attention to it. None of the other players noticed it.Jake Henry by email.

A The Laws have not changed and the directors were wrong.

Once the player has led to the next trick the revoke is established, the hand is played out and two tricks are awarded to declarer (only one if the defence made no more tricks after the revoke trick). Perhaps your directors should get in the habit of reading rulings from the Law book.

Dummy is not allowed to point out a revoke until the end of the hand, but may speak then.

♣♦♥♠

QRecently after playing the last but one round

of the evening, it was discovered that the cards had been misboarded by the previous table. They had been rotated by 90 degrees so that West played South’s cards, etc. The director corrected the board for the final round to be played. When the corrected results were displayed, it appeared that the director had adjusted our score so that E/W who played the hand were given the N/S position and scored accordingly for that round.

For future reference, what should happen when hands are not returned to the correct position?Sally John by email.

A It was certainly a completely illegal decision. When you

played the board the cards were in a different position with a different dealer and it is not possible to compare the score on the

BRIDGE April 2016 Page 23

board with anyone else. The board should have been cancelled, both you and your opponents should get average plus (usually 60%) since presumably you could not tell it was wrong so it was not your fault, and both pairs at the previous table should be penalised, and lectured on leaving the board in the middle of the table, since that is the only likely reason for this happening.

♣♦♥♠

QMy partner bid 4NT, Blackwood. I bid 5♦ to show

one ace. My partner then bid 5NT, which I passed as we play progressive ie 5♥ would have asked for kings. An opponent requested information and then called for the director. The experienced director listened, said play on and come to me if anyone is unhappy. It was a Sim Pairs and pressure was on time and we were late for the next table. My partner made 10 tricks, 16 pairs made either 11,12 or 13 tricks. Looking at the hands later, it transpired that the opponent who called the director had a flat hand with zero points and got a top score of 34 points. He is known to have a habit of calling the director, which generally sends a frisson to players, puts pressure on opponents especially with time constraints.

There is the suspicion that some players can thereby intimidate opponents, who may be inexperienced or are playing with a new partner or a new system etc, by calling the director. In my opinion, this tarnishes bridge, puts players off duplicate and undoes the good work some clubs have of emphasising ethical behaviour, courtesy, etiquette and politeness etc. Do you think that my opponent was correct in calling for the director, not having to lead, having zero points and a flat hand?Leslie Phillips by email.

AOf course, a lot depends on the director doing the

correct thing. I emphasise calling a director when something has gone wrong. In this case, there is no evidence whatever that anything has gone wrong, and as a director I would have warned your opponent for time-wasting and possible intimidation.

The hand he holds is irrelevant: the question is whether there is any potential irregularity, but I can think of none. However, it is important that players do not feel calling the director causes any problem: players should accept it as part of the game, no more surprising than when a football referee blows his whistle and indicates a corner-kick. No rules have necessarily been broken, and even if they have, that is not a hanging offence! ■

Ask David continued

E-mail your questions (including your postal address) on bridge laws to: [email protected]

DEFENCEQUIZ

by Julian Pottage (Answers on page 24)

You are East in the defensive positions below playing matchpoint pairs with both sides vulnerable. Both sides

are using Acol with a 12-14 1NT and 2♣ Stayman.

NW E

S

1. ♠ 6 4 ♥ K J 7 6 4 ♦ K J 7 ♣ Q 8 2 ♠ J 10 7 2 ♥ 10 9 3 2 ♦ 8 4 ♣ A J 5

West North East South 1♦ Dbl 1♥ 1♠ 2♦

2♠ 3♦ All Pass

Partner leads the ♣10. What is your plan?

NW E

S

2. ♠ Q 10 4 ♥ 4 ♦ A Q 8 7 3 2 ♣ K Q 8 ♠ 7 2 ♥ 10 6 3 ♦ K 10 4 ♣ A 9 7 5 2

West North East South 1♠ Pass 2♦ Pass 2♥

Pass 4♠ All Pass

Partner leads the ♣10. What is your plan?

NW E

S

3. ♠ 10 8 7 2 ♥ J ♦ A J 3 ♣ 9 6 4 3 2 ♠ Q ♥ K 10 6 3 ♦ K 9 4 ♣ A J 8 7 5

West North East South 1♠ Pass 2♠ Dbl Pass 3♥ 3♠ All Pass

Partner leads the ♣10. What is your plan?

NW E

S

4. ♠ K J ♥ 10 9 8 2 ♦ A J 8 2 ♣ Q 6 2 ♠ A 5 4 2 ♥ A J 7 ♦ 7 ♣ A 8 7 5 3

West North East South 1♣ 1♠ 2♣ Dbl 3♣ 3♠

Pass 4♠ Dbl All Pass

Partner leads the ♣10. What is your plan?

Page 24 BRIDGE April 2016

Answers to Julian Pottage’s Defence Quiz on page 23

NW E

S

1. ♠ 6 4

♥ K J 7 6 4

♦ K J 7

♣ Q 8 2

♠ A Q 9 5 ♠ J 10 7 2

♥ A Q 5 ♥ 10 9 3 2

♦ 9 5 ♦ 8 4

♣ 10 9 7 6 ♣ A J 5

♠ K 8 3

♥ 8

♦ A Q 10 6 3 2

♣ K 4 3

West North East South

1♦

Dbl 1♥ 1♠ 2♦

2♠ 3♦ All Pass

Partner leads the ♣10. What is your plan?You cannot be sure whether the lead

is from an interior sequence or a ten-high suit. In any case, by saving the ♣A to capture the ♣Q, you minimise your opponent’s tricks in the suit. When the ♣10 loses to the ♣K, you know the club position.

The second club lead will come from partner and you finesse the ♣J. You can then switch to the ♠J (probably best to do so before taking the ♣A, in case declarer started with a doubleton club). You anticipate that partner has a tenace in spades to explain the initial lead.

NW E

S

2. ♠ Q 10 4

♥ 4

♦ A Q 8 7 3 2

♣ K Q 8

♠ K 6 5 ♠ 7 2

♥ A 9 7 5 2 ♥ 10 6 3

♦ J 9 5 ♦ K 10 4

♣ 10 6 ♣ A 9 7 5 2

♠ A J 9 8 3

♥ K Q J 8

♦ 6

♣ J 4 3

West North East South

1♠

Pass 2♦ Pass 2♥

Pass 4♠ All Pass

Partner leads the ♣10. What is your plan?Since you can see the ♣9, you know

that the lead is from a shortage. Do you think it is a singleton or doubleton? Some-times the mere fact that partner chooses to lead the suit will incline you to think it is a singleton. Here the only unbid suit is clubs, so you cannot apply that inference. Indeed, as South has shown at least five spades and at least four hearts in the bid-ding, the chance it is a singleton is slim.

The way to give partner a club ruff is duck the first trick, playing the ♣7 or even the ♣9 to encourage. Your partner will soon gain the lead, either with the ♥A or the ♠K, and be able to continue clubs. Taking the first club would fail because you never get in again.

NW E

S

3. ♠ 10 8 7 2

♥ J

♦ A J 3

♣ 9 6 4 3 2

♠ J 9 5 ♠ Q

♥ A 8 7 5 2 ♥ K 10 6 3

♦ 10 8 5 2 ♦ K 9 4

♣ 10 ♣ A J 8 7 5

♠ A K 6 4 3

♥ Q 9 4

♦ Q 7 6

♣ K Q

West North East South

1♠

Pass 2♠ Dbl Pass

3♥ 3♠ All Pass

Partner leads the ♣10. What is your plan?Again, you can see the ♣9 and so

know that the lead is from a shortage. This time, since partner would not lead the ♣10 from K-10, Q-10 or K-Q-10, you know it is a singleton.

For sure, you want to take the ♣A and give partner a club ruff. After that, you are in no rush for a diamond switch. The ♦K should not run away. You would rather like to get in with the ♥K to play a third round of clubs. With the singleton heart in dummy, you need to get partner to underlead the ♥A. You should give a suit-preference signal with the club you return. You lead the highest club you can afford, the ♣8, to ask for the higher red suit. Let us hope partner trusts you.

NW E

S

4. ♠ K J

♥ 10 9 8 2

♦ A J 8 2

♣ Q 6 2

♠ 8 ♠ A 5 4 2

♥ 6 5 4 ♥ A J 7

♦ Q 10 6 5 4 ♦ 7

♣ K 10 9 4 ♣ A 8 7 5 3

♠ Q 10 9 7 6 3

♥ K Q 3

♦ K 9 3

♣ J

West North East South

1♣ 1♠

2♣ Dbl 3♣ 3♠

Pass 4♠ Dbl All Pass

Partner leads the ♣10. What is your plan?Expecting partner not to have raised

you with three clubs to the ten, you read the lead as from an interior sequence. You must put up the ♣A to stop the singleton ♣J from scoring. Should you switch to your singleton diamond?

If partner gains the lead, you will score 200 and a top (or near top) even without a ruff. With your four trumps, you do bet-ter to play a forcing game. If you keep playing clubs each time you get in with an ace, you should end up with a long trump after declarer keeps ruffing. You just need to consider one more point. You want to ensure that declarer has to ruff in hand, so you should hold up the ♠A. ■

BRIDGE April 2016 Page 25

The theatre trip had been a great success. Ibsen’s ‘A

Doll’s House’. The acting was superb. Awesome in fact for a local company. Some of the group hadn’t wanted to go because they thought it might be a little depressing. Mary was just glad of the opportunity to belong to such an outgoing and vivacious group. There was always something going on. She loved being part of the group. This was all new to her. When Peter was alive they really had been very insular. They hadn’t needed or wanted anyone else. But now, she was really grateful for the new friendships being made.

West North East South

1NT1 Pass 2♣2 Pass

2♦3 Pass 3NT All Pass112-142 Stayman 3No 4-card major

NW E

S

♠ A 5 ♠ K Q 4

♥ K 6 3 ♥ A 7 4 2

♦ Q 8 6 2 ♦ K J 5

♣ K J 4 3 ♣ Q 6 5

West is in 3NT. The ♠J, top of sequence, is led by North. West counts her top tricks. Three spades, A-K-Q, two hearts, the A-K. Total five. Four tricks short. Oh dear! West is feeling the pressure, forgetting everything that Alasdair has said. She

panics and immediately plays out her five winners. Now what? The game is played amazingly quickly with West going down two. ‘Sorry partner.’

The MoralAfter counting the five top tricks West must stop, think and plan where the extra four tricks will come from whilst she has stoppers in the other suits. The stoppers are the ♠A-K-Q and the ♥A-K. The stoppers can allow West to set up and establish four winners. Whilst no-one encourages slow play, declarer is allowed to take the time to plan the play. Study both the diamond and club suits. Can you see a pattern? Don’t leave these suits until you recognise how easy it is to set up two tricks in each of the minors. Yes, once the ace is knocked out by the king in both suits, declarer can set up the queen and jack as two winners in each suit. As soon as West wins a trick in either hand she must set about knocking out the ace in one of the minors, establishing two extra tricks. When declarer gets in again, she must knock out the second minor suit ace, establishing the third and fourth extra tricks necessary to make the contract. Only then can West play out her remaining stopper winners in the other suits. Nine tricks and contract made.

Beginners’ Bridge Corner

Sorry Partner! Stop, Think, Plan

by Liz Dale You are sitting in the West seat with this hand:

NW E

S

♠ 7 2

♥ 10 6 2

♦ 7 6 3

♣ 10 8 5 4 2

You are not expecting to have to do much and get ready for a little nap, whilst the other three players play out the hand. It is when you hold such hands that your place in your partnership can be secured. You should be thinking: what can I do for my partner. Is there any way I can make his life easier? Clearly, he is going to have to do most of the work during the auction and the play. You are likely to end up defending, so perhaps you can make a difference by choosing the best lead for the partnership. Note, the best lead for the partnership, not for just your hand.

West North East South

1♠ Pass 1NT

Pass 3NT All Pass

You hear the auction and need to make your lead.

Your aim is to either find your partner’s suit or at least lead through some of dummy’s strength. Your partner is going to be on lead a lot of times if you are going to have a chance of defeating the contract. Rather than choosing your longest suit, choose a suit that has not been bid, but you are

not so long in. I would choose between hearts and diamonds and generally if I am in doubt between an unbid major or minor, I tend to go for the major.

Which heart should you lead?

You probably go to the text book and look up the lead, but better is to think about your ambitions. This is your one and only chance to be on lead, so surely it is best to lead a higher card to try and force dummy’s play. Lead the ♥10.

NW E

S

♠ A K 10 3

♥ K 9 3

♦ A 9 8

♣ A J 6

♠ 7 2 ♠ Q 9 6 5 4

♥ 10 6 2 ♥ A Q J 4

♦ 7 6 3 ♦ K 4

♣ 10 8 5 4 2 ♣ Q 7

♠ J 8

♥ 8 7 5

♦ Q J 10 5 2

♣ K 9 3

Since I set this, the lead works like a charm allowing your side to take the first four hearts. Your partner also makes the ♦K. Notice that a low heart would have been ducked in dummy and left our partner on lead allowing declarer home. Any other lead gives declarer plenty of time to establish nine tricks since East will be the only defender to get the lead.Most declarers will be making 3NT, but it was the player with no points at all that had the opportunity to defeat it.

When you are very weak try to think of your partner

More Tips from Bernard Magee

Page 26 BRIDGE April 2016

The summer of 2015 will be re-membered for the dramatic revelations that top interna-

tional players were alleged to be con-veying information to their partner with the pre-planned and deliberate goal of achieving better results.

Some of you will remember that back in March 2014, we learned of the famous ‘German Doctors’ case, and how a seemingly unlikely pair were found to be communicating via coughs and gestures. The media loved drawing parallels with ‘Who Wants To Be A Millionaire?’ and the use of a cough or two.

Such is the law regarding libel, that your author is taking great care not to repeat conjecture, or step out of line in any way. The internet is another matter altogether, and policed by different authorities, or possibly by no authorities. I am reliably informed that libel is actionable online too – and the same legal authorities would be involved – though there are clearly issues around how it is enforced and who enforces it. So do take a moment to look at the links if you want to know

Globe Trotter Part Two

Not playing the game in accordance with the rules. Despicable.

that the very strict rules concerning the shaking and throwing of dice, combined with the lipped design of the cups used, means that it is very difficult to cheat. He added that para-noid players insist on the use of a ‘dice

tower’, which was used by Romans when they played backgammon. The internal ridges and steps serve to ran-domise dice rolls.

Refer to Wikipedia for more: Vettweiss-Froitzheim Dice Tower.

Scrabble‘Last week, there was a dustup over a teen caught cheating at the National Scrabble Championship. The news was everywhere. Like a nun smoking a Marlboro, the clash of branded rectitude with fiendish perfidy struck a scintillating chord.’ David Bukszpan, Huffington Post, August 2012.

How? He had hidden blank tiles up his sleeves. This plan was perhaps taking simplicity too far. There are only two blank tiles in the game of Scrabble. Yes, he had an advance tile ‘up his sleeve ahead of time, ready to

more about the above-mentioned episodes, or look back at Ned Paul’s article in BRIDGE 155, page 16.Newsweek (23 Sept 2015)http://tinyurl.com/gp9hxltThe Telegraph (29 March 2014)http://tinyurl.com/nf8gpv6

♣♦♥♠

Setting aside the sweeping allegations about dodgy practices in the Olympics, football, athletics and tennis arenas, I thought it would be interesting to know what recent scandals some other games have had.

BackgammonHugely popular because of the internet, and very accessible. It is easy to get up and running, and only two players are needed.

But what about cheating? Is it easy? In face-to-face backgammon the shady player might try to use loaded dice, influence the roll of the dice, or move pieces whilst the opponent is not looking. Playing online, it is possible to use a computer program to help you with the moves. Because the game has simple rules, and relies on facts known to both players, at all times (ie unlike bridge, where we have to suppose a distribution or holding in a suit, based on bidding or play), the programs can come up with mathematically expert plays in fractions of a second.

So can online backgammon cheating be combatted? Interestingly the online sites utilise move-comparison software to try and identify situations where a player is playing in a style that is too similar to a program.

Michael Crane, British Isles Back-gammon Association (BIBA) Founder and Tournament Director, tells me

Millionaire story update

Former army officer Charles Ingram published a book in January 2015 to back up his claim of innocence. On TV in 2001 he had won the jackpot, but a court in 2003 ruled that his win was invalid. Rory Sutherland wrote about this subject in his column in The Spectator, in February 2015.http://tinyurl.com/jxxfj5r

Photo: Jürgen Vogel. LVR - LandesMuseum Bonn

BRIDGE April 2016 Page 27

be used’, but what on earth would he do if his opponent drew two blanks subsequently? And how about the possibility that he himself would draw the two genuine ones? How will he dispose of (or swap) the now-burdensome spare?

There is no bridge equivalent to this caper. In most games you cannot expect to get away with producing an extra ace of trumps, because one of the other players will usually notice that he also holds this card.

You will, of course, have heard of poker players having cards up their sleeves. They are considerably less likely to be detected in private games.

Returning to Scrabble, you are allowed to play a word that isn’t really a word. Bukszpan explains that in

Globe Trotter continued order to counter the risk of being challenged (remember that there are regional official variations, and of course, when played at home, ‘house rules’), players quickly replenish their rack to up the tempo of the game. The bridge equivalent would be that if you know you have revoked, you claim quickly and score up, and play the next board.

Yes, strange as it may seem, in bridge you don’t need to own up to your own revoke. Of course, you are required to follow suit when you can, and a deliberate attempt to not do so is very wrong.

ChessDubai, 17th annual Dubai Open Chess Tournament, April 2015. The media widely reported the case of Georgian Grandmaster Gaioz Nigalidze, a 25-year old, who allegedly made frequent trips to the toilet, and took the opportunity to use his smartphone which had a chess program.

Just as in Backgammon, the advances in the power of computing means that such a concealed aid can be hugely beneficial.

The upshot was that he was expelled from the tournament, and FIDE ruled in December 2015 that he was to be banned for three years. His grandmaster title has been revoked. The judgment includes ‘…his international master (IM) title, obtained already in 2009, was left intact in recognition of his remorseful and cooperative conduct in the investigation.’

Nigalidze was not the only person to have acted like this, nor was he the only person to be caught. But he is amongst the most accomplished and infamous.

♣♦♥♠

So looking ahead, it will be interesting to see how many of the bridge players accused of wrongdoings in 2015 will be able to clear themselves in court, or in the much more punishing court of public opinion (tougher, lower standards of proof, prone to speculation and sometimes ill-founded deductions). Watch this space. ■

The most comical accusation?

The 1978 World Chess Championship

The match of 1978 deserves its reputation as the most bizarre World Championship match ever played. Karpov’s team included a Dr. Zukhar (a well known hypnotist), while Korchnoi adopted two local renegades ... There was more controversy off the board, with histrionics ranging from X-raying of chairs, protests about the flags used on the board, the inevitable hypnotism complaints and the mirror glasses used by Korchnoi. When Karpov’s team sent him a blueberry yogurt during a game without any request for one by Karpov, the Korchnoi team protested, claiming it could be some kind of code. They later said this was intended as a parody of earlier protests, but it was taken seriously at the time. Source: Wikipedia

Fly to Athens to join Aegean Odyssey • Cruise to Gythion (Sparta & Mystras)

• Katakolon (Olympia) • Ithaca • Corfu • Sarande (Butrint) • Kotor Bay

• Dubrovnik (overnight onboard) • Split • Zadar • Ravenna • Venice

• Fly home

Enjoy 8 included sightseeing toursand daily duplicates on this

fascinating bridge cruise from classical Greece to Venice

Great value Mr Bridge fares

Standard Inside from £1,795pp

Standard Outside from £2,250pp

Premium Outside from £2,395pp

Single supplement just 10%

May 11, 2016 - 10 days from Athens to Venice

www.mrbridge.co.uk

For reservations call

on 01483 489961

Terms and conditions apply – see brochure or website for details

Greece& the

Adriatic Seahosted by

SHEILA ROGERS and GWEN BEATTIE

2101 Mr Bridge third page ad 160511BR_Layout 1 2

Page 28 BRIDGE April 2016

When players meet and discuss their agreements, they tend to spend a lot

of time on the bidding and very little on defending. This article covers agreements at trick one. You need agreements as to what to lead and how to signal.

Honour leadsWhilst your decision as to what to lead is often based on what you have heard in the auction, when you have decided on the suit to lead you need an agreement with your partner as to which card you will lead. If you lead the queen in a suit then partner is likely to expect the jack and not the king if you have standard agreements. If you lead low he is likely to expect fourth highest in the absence of any other agreement. If he is a thinking defender, he will use this information during the hand along with other evidence that piles up.

Let’s start at the top. What are your agreements leading from a suit headed by honours? It is normal to agree to lead the top honour from a sequence, so queen from Q-J and jack from J-10 and 10 from Q-10-9. With suits headed by the A-K, some players will lead the ace and others the king. Some will lead the ace if they would like an attitude signal and the king if they would like a count signal. Imagine the opponents have bid quickly and uninformatively to 6♠ and you hold:

NW E

S

♠ 7 6 5

♥ 4 3 2

♦ A 9 6 5

♣ Q 7 3

You decide to lead the ♦A and dummy goes down with:

NW E

S

♠ K 9 5 4 2

♥ A K 7

♦ Q 6 3

♣ 4 3

What do you play at trick two? This is, of course, a ridiculous question as it depends on what partner plays. What would you like him to play? If you lead an ace against either a slam or a pre-empt, then what you most want partner to do is to tell you whether he likes it. You usually won’t have A-K in a suit. I’m sure the opponents will have bid badly to reach 6♠ missing two aces or the cashing ♦A-K, but the prime responsibility is to punish them if they have. If partner plays a high card then continue with diamonds. If he plays a low card then try another suit (a club on this hand). If you did have an A-K against a slam, you would lead the king and expect partner to give you count so you know whether declarer really might have two immediate losers.

To make life more complicated, the signal partner gives depends not only on your lead but also what he can see. If you lead the king, you are expecting a count signal but suppose your lead is the ♦K from ♦A-K-9-6-3 and the dum-my, in a contract of 4♠, comes down with the hand in the next column.

What do you play at trick two? The answer, of course, is that it depends. You have agreed that if you lead the king, partner gives count.

NW E

S

♠ A Q 7 5

♥ Q 6 4 2

♦ 7

♣ Q 6 4 2

Of course, on this hand, count in diamonds is unlikely to be very useful so what might you do instead? A lot of players would give suit preference here so that a high diamond suggests a heart switch and a low diamond suggests a club switch.

Here is another example whereby you have to make a sensible choice. You lead the ♥K from ♥K-Q-8-3 against 4♠ and the dummy has:

NW E

S

♥ A 7 6

♥ K Q 8 3

Declarer ducks the king and you are still on lead. Your agreement with partner is he shows you count on the lead of the king so what do you do? Declarer has probably ducked specifically to make you guess at trick two what is best. If you have an agreement with partner that you will give attitude for the jack (signalling high with the jack and low without) then you will not continue and give declarer a cheap trick.

It’s beginning to sound as if there are more exceptions than rules. The vast majority of the time, you will give the

Conventions Part 16 with Jeremy Dhondy

Trick One

BRIDGE April 2016 Page 29

signal you have agreed on, but you do need a little flexibility if you are to get as many situations right as you can.

If you find counting hands exhausting, then play more attitude and less count.

Against no-trumpsYou are quite likely to lead an honour against a suit contract, especially with a sequence, but against no-trumps where the main aim is to develop a suit so that you have winners with low cards later in the play, a standard agreement is to lead fourth highest from length. Most players do this, but I wonder how many know why or what to do with the information.

If you hold ♠ J-8-6-5-2, why do you lead the ♠5 and not the ♠2? The reason for this agreement is so that partner can best work out what you have. A simple example is:

NW E

S

Dummy

♠ 10 9 6

Partner You

♠ 4 ♠ K J 5

If you deduct partner’s card from 11 you are left with 7. Dummy holds 3 cards higher than the four as do you. That leaves only one for declarer (11-4-3-3=1). If you are counting points, you now know that at least two and possibly four, but not more, of declarer’s points are in spades. As the hand goes on you will be able to add more evidence to help you.

Leading from only small cardsWhen partner led a low spade, you also knew that he had an honour because another standard agreement is that if you have only low cards in a suit, you lead the second best with three or more, so from 8-7-6-3 or 8-7-3, you lead the 7. If you lead this against a suit contract it can help partner in a situation such as:

NW E

S

♠ A J 5 4

♠ 8 7 6 3 ♠ K 10 2

♠ Q 9

If you lead the 3 (fourth highest), partner would expect an honour and insert the 10. In a suit contract, your winner would just have evaporated. With the lead of the 7, partner knows to rise with the king. Mind you, he still doesn’t know if you have three or four and your second card should tell him this. With three cards, you play the top card next (MUD or Middle up down). With four or more small cards, you play the original fourth highest next, so that if you play the 7 then the 3 in this situation, partner knows you have four (or, more unusually, two). With two low cards you lead the higher. This is standard although beware if you ever go to Poland as standard is low from, say, 9-3.

How about the 10?Is the 10 an honour? Players like arguing about this and it may affect what you lead. If the 10 is an honour then you lead the 2 from 10-8-4-2. If it isn’t, you lead the 8. On most hands it will not make a difference but you should agree with your partner. If you disagree with him then your clinching arguments (depending on what you prefer) are that partner will expect more than the 10 if you lead a low card or that the lead of the 8 will destroy a useful holding at trick one. You pays your money . . . Whatever you agree, you can rule out some holdings which may aid your defence.

Sometimes, of course, your side will have taken part in the auction. If partner has bid a suit, then you need a good reason for leading something else unless your specific wish is to annoy him (having a void in his suit is an acceptable reason). What do you lead? The usual answer is that you lead whatever you would have led in the suit if he had not bid. Suppose he had made an overcall of 1♠ and you had raised him to 2♠ and your hand

Trick One continued included either ♠8-3-2 or ♠K-8-3.What would you lead? Well, MUD

from the first and low from an honour on the second. The problem is, as you can no doubt see, that partner will have to guess. One solution is if you have supported, then partner will expect three or more cards in the suit, so you lead the ♠8 from ♠8-3-2 and a low card from the holding with an honour. Partner would never have expected you to support on ♠8-x, so there is little danger of misleading him.

If you have some agreements with partner as to what you will lead and how you will signal, you can avoid some common misunderstandings. As you become more advanced, you will be able to count the hand much better and therefore improve your side’s defence. ■

Summaryl Agree not only what you will

lead, but also what signal you want partner to give at trick one.

l If you win trick one and you have led a suit where there is a singleton in dummy, consider agreeing with your partner that his card is suit preference.

l Make an agreement as to what signal you will give at trick one (perhaps attitude on the ace and count on the lead of other honours), but be aware that there are a few sensible exceptions (the singleton in dummy and the avoidance of giving a cheap trick to the jack are two).

l If your partner leads fourth highest against no-trumps, then use the rule of 11 to establish what is going on in the suit.

l Lead second best from three or more small cards to help partner know whether you have an honour or not.

l Agree with partner whether the 10 is an honour or not.

l Consider leading top from three small if you have supported your partner in the auction.

Page 30 BRIDGE April 2016

QI was North playing matchpoint pairs

with a regular partner.

NW E

S

♠ Q 8 6

♥ K J 3 2

♦ 4 3 2

♣ K 5 4

♠ J 5 2 ♠ 10 9 7 4 3

♥ A Q 9 8 ♥ 10 6

♦ K Q J ♦ A 10 9 8 7 6

♣ 6 3 2 ♣ Void

♠ A K

♥ 7 5 4

♦ 5

♣ A Q J 10 9 8 7

West North East South

1♣

Dbl Rdbl 1♠ 3♣

Pass 3NT All Pass

On my rebid, I thought that 3NT would be a better punt than 5♣. It seemed that spades were the danger suit; East led diamonds and the opponents took the first six tricks. As it happens, 5♣ is the only makeable game. Four pairs out of ten in the room bid 5♣; all others stopped in a part score. Where did we go wrong? We play Benji,

so an opening bid of 2♣ by South was possible. Ray Burnett by email.

A 1. Vulnerable against not with an eight-loser hand,

West’s take-out double is aggressive. East’s cautious 1♠ (with a 7-loser hand) suggests a familiarity with West’s style.

2. I do not know why North would redouble. An awful nine points – 4333 shape and appalling intermediates – is not enough. Moreover, redoubling suggests a desire to penalise the opponents.

3. After North’s redouble, an immediate action from South, even a jump, suggests minimal values (though the jump does indicate a long club suit). South should pass (forcing). North would then reopen with 1NT and South would then jump to 3♣.

4. Having overbid with the initial redouble, North should pass 3♣.

Now suppose the auction starts 1♣-Pass-1♥. South has a tricky rebid. In terms of playing strength, 3♣ is a slight underbid. It also makes it hard to find a 5-3 heart fit. However, with only 14 HCP and long clubs, nothing else but 3♣ really appeals either.

North probably passes 3♣.Even with the perfect

holding in diamonds, three small facing the ace, 5♣ still needs a favourable heart position. I do not think you should worry too much about missing a thin game at matchpoints.

With 14 HCP, a strong opening by South would be unwise, despite the eight clear-cut tricks.

♣♦♥♠

QWhy, when you have a 5-3 fit plus a 4-4 fit in

the majors, is it better to play in the 4-4 fit?Bill Hogan, St Helens.

A There are two main reasons why a 4-4 fit usually plays

better than a 5-3 fit:1. Playing in a 4-4 fit, a

ruff in either hand generates an extra trump winner; in a 5-3 fit, only a ruff in the short trump hand generates an extra trick.

2. In a 5-3 fit, if trumps break 4-1, you lose control if you have to ruff twice with the 5-card holding; in a 4-4 fit, ruffing twice in the same hand still leaves four

Julian Pottage answers your bridge questions

How Should we Have Bid to the Correct Game Contract?

trumps in the other hand.Consider these two hands:

Hand 1 Hand 2

♠ K J 5 2 ♠ A Q 10 7

♥ K Q 10 8 2 ♥ A J 4

♦ A 5 3 ♦ 9 6 2

♣ 5 ♣ A 9 2

Playing in hearts, the 5-3 fit, you cannot make even a small slam. You have two diamonds to lose. By contrast, if you play in the 4-4 spade fit you stand an excellent chance of being able to ruff two clubs in the West hand to make all thirteen tricks.

Consider also these two hands:

Hand 3 Hand 4

♠ K J 5 2 ♠ A Q 10 7

♥ K Q 10 8 2 ♥ A J 4

♦ Q 5 3 ♦ J 10 2

♣ 5 ♣ J 9 2

If the defenders start with two rounds of clubs, a 4-1 heart break is likely to defeat 4♥. Declarer cannot make ten tricks without setting up a diamond trick and will be subject to a further club force when the defenders

BRIDGE April 2016 Page 31

get in with their diamonds. Conversely, in 4♠, having to ruff a club in the West hand does not inconvenience declarer in the slightest even if spades are 4-1.

As you have asked about the majors, I have given you examples involving the majors. At slam level, you generally want to play in whatever suit gives you the best chance of making the contract, whether it is a major or not.

♣♦♥♠

QYou are playing in 4♠ and need to bring

in the following trump combination for only one loser.

NW E

S

♠ A 6 4 3

♠ Q 9 7 5

1. With no other information, what is the best way to play the suit?

2. You decide that West is most likely to hold three spades; does this change your play?

3. If you lead low from the North hand, East plays the eight.Simon Gottschalk,Pendoylan, Mid Glamorgan.

A Suitplay is a great piece of software for analysing

suit combinations.http://home.planet.

nl/~narcis45/SuitPlay/1. Without any clues

about the layout, it seems normal to start with the ace.

If the king falls on your left, you then lead twice up to the queen-nine. If the ten or eight falls on your left, you finesse the seven.

2. If you decide that West is most likely to hold three spades, you start low rather than with the ace.

3. When East then plays the eight on the first round, this raises an interesting possibility. You can play East for 10-8 or J-8 rather than K-8 (and some defenders would play the king from K-8). You cover with the nine (a play known as an intra-finesse) and run the queen next time.

♣♦♥♠

QI was North, dealer, playing matchpoint pairs:

NW E

S

♠ K J 10 2

♥ A K 5 4

♦ J 6

♣ A 8 6

♠ A Q 9 8 7

♥ 8 7

♦ 9 5

♣ K 5 4 2

I opened 1♥. My partner responded 1♠ and I raised to 2♠ with my seven-loser hand, despite having 16 points. He passed and we missed game. Should I have gone 3♠ or should my partner have bid game?Martin Epstein by email.

A If both of you used the losing trick count, you would

reach game because you have a pair of seven-loser

hands facing each other. You seem to have missed the game because one of you was using losers and the other point count.

With your hand I probably would raise 1♠ to 3♠. Apart from the doubleton jack, which might be waste paper, everything else about your hand looks good. You have nice trumps and fast winners outside.

With your partner’s hand, while I would not bid 4♠ over 2♠, I would make a game try of 3♣. This middle action seems reasonably clear. With a seven-loser hand and five good trumps, passing 2♠ is excessively cautious. With only 9 HCP, a jump to 4♠ is overly ambitious.

♣♦♥♠

QIn a teams match, my partner finished

in 6♥. Cue-bidding had established we were missing first-round control of clubs, so it came as no surprise that West led the ace of clubs. What did surprise us was that East showed out, thus seemingly strangling the contract at birth. Naturally, West followed with a low club and we waited for the inevitable ruff from her partner. To our amazement East discarded, being void in both clubs and hearts! None of us at the table could ever recall being dealt a hand with two voids and apart from saying the odds must be astronomic we were unable to be more specific. Could you enlighten us please? The contract, by the way, went on to make.Eddie Blount, Loughborough.

A Yes a hand with two voids certainly is unusual.

The chance of picking up these hand types with two voids is as follows:

7-6-0-0 .0056%8-5-0-0 .0031%9-4-0-0 .0010%10-3-0-0 .00015%

These total roughly .00985%, in other words a fraction less frequently than one hand in every 10,000. (I have ignored the shapes more extreme than these because they are so rare.)

♣♦♥♠

QI was sitting North, playing matchpoint

pairs with only our side vulnerable.

NW E

S

♠ K

♥ Q J 7 3

♦ K 8 4

♣ A Q J 6 4

♠ A Q

♥ 2

♦ A Q J 9 7 2

♣ K 9 7 2

West North East South

3♠ Dbl

Pass 4♣ Pass 5♣

All Pass

I made 13 tricks, although on a heart lead I would be held to 12 tricks. My partner told me that my 4♣ was wrong while I thought she should have overcalled 3NT. Incidentally, East had opened 3♠ with ♠J-x-x-x-x-x and a bust.Flora Simons, Onchan, Isle of Man.

Ask Julian continued

Page 32 BRIDGE April 2016

AOpposing pre-empts do make your life difficult. This is why

they are made. One of my teammates opened 3♥ with a jack-high six-card suit in the Camrose, so this does happen.

Your partner was partly right. It is usual when someone comes in over a pre-empt to assume 8 points opposite. With your 16, you must bid game, 4♥ or 5♣. Having underbid with 4♣, you should have raised 5♣ to 6♣.

Yes, with a double spade stopper and an inability to cope with a 4♥ reply to a double, partner should have bid 3NT. With limited options over 3♠, a 3NT overcall does not promise a balanced hand. 4♦ would be my second choice. Doubling with a singleton in an unbid major is very risky.

Over 3NT, 4♣ from you would be a slam try – one does not rescue a game contract to a part score. Your partner has a great hand in support of clubs and would either bid 6♣ directly or make some sort of cue bid. Either way you should reach the slam.

♣♦♥♠

QOver a 2♣ opener (23 plus or 8 plus tricks) my various

partners and I play responses as follows:

2♦ 0-3 points2♥ 4-6 points2♠ 7-9 points2NT 10+ points

We find that we have a pretty good idea as to whether we are looking at a game, slam or possibly a part-score as the final contract.

It seems to us to be very useful but nobody else appears to use it or recommend it.

What do you see as the pros (if any) and cons?John Ruddock, Kibworth, Harcourt, Leicestershire.

A Although the particular method you describe is

indeed unusual, other people do or at least have played something similar. The CAB system that was popular some 50-60 years ago used ace-showing responses to a 2♣ opening. The Blue Club system, as used by the multiple World Champion Italian Blue team, used control-showing responses to a strong 1♣ opening.

You are quite right that the consensus view now is to show shape first and strength later. Showing the shape first certainly helps on a competitive auction. If opener has two suits to show, it helps if responder initially keeps the bidding as low as possible. Often what matters is not how strong responder is but whether the values are in the right place.

Given your preference for showing strength first, the ranges you have chosen look sensible. The 0-3 response tells the partnership that a slam is very unlikely and even a game may not be possible. The 10+ response puts the partnership straight in the slam zone. ■

Ask Julian continued

E-mail your questions (including your postal address) for Julian to: [email protected]

NW E

S

1. ♠ A 7 5

♥ J 6 4

♦ 9 7

♣ K Q 7 3 2

♠ 6 4

♥ A 5 3 2

♦ A K 6

♣ A 10 9 8

You are declarer in 3NT and West leads the ♠K. How do you plan the play?

NW E

S

2. ♠ 9 5 4 2

♥ A 6

♦ K J 5 4

♣ 8 4 3

♠ A 6 3

♥ K Q J 7

♦ A 6 3

♣ K J 5

You are declarer in 3NT and West leads the ♣6 with East playing the ♣10. How do you plan the play?

NW E

S

3. ♠ Q 6 4 2

♥ A Q 7

♦ J 9 7 5 3

♣ 2

♠ A J 9 7 5 3

♥ K 10 2

♦ 2

♣ A K Q

You are declarer in 6♠ and West leads the ♦A, ♦K. How do you plan the play?

NW E

S

4. ♠ K J 6 4

♥ J 7 3

♦ A J 2

♣ A J 2

♠ A Q 10 8 7

♥ 9 2

♦ K 6 3

♣ K 10 7

You are declarer in 4♠ and West leads the ♥K followed by the ♥4. East wins the ♥A and returns the ♥6. How do you plan the play?

DECLARER

PLAY

QUIZby David Huggett

(Answers on page 34)

You are South as declarer playing teams or rubber bridge. In each case, what is your play strategy?

BRIDGE April 2016 Page 33

Teacher’s Corner – Teaching Tips from Ian Dalziel

Bidding Tents

A lot of the teaching of bidding involves the uncontested auc-

tion. After the spoken les-son, students usually prac-tise what they have been taught with prepared hands. The problem is that only one side is bidding and the other side is bored waiting their turn to bid on the next hand; if it’s a long slam auc-tion the other side might fall asleep! Then the hands are played out which can take more time than the bidding. Hence it will take 10 to 15 minutes per board, includ-ing the questions and dis-cussion at the end.

Even if eight boards are played at the class, each pair will only be bidding four hands. If the lesson is about responding bids, each player will only ‘respond’ twice in the session. The best way for the student to learn is lots of practice, so this is not a very efficient use of the time.

My solution is to have them working in pairs us-ing bidding tents. A bidding tent consists of a folded card with a hand on either side as shown (just like a tent). The players bid the hand to conclusion (assuming oppo-

nents are silent) and write their bidding on a special pad (see later). When bid-ding is over, they open the tent and the two hands are shown inside with the rec-ommended bidding and explanation. They check their bidding and, if they are wrong, they can see why. There are 20 tents in a set stacked on top of each other. When they finish one tent they put it at the bot-tom of the stack and bid the tent now on top; the dealer alternates.

The HCP are shown on the outside and inside of the tent. This saves time and prevents errors in count-ing points. The teacher is on hand to help, but is rarely needed – they teach themselves. The discussion is incredibly helpful; the students love it as no-one is inactive or bored. 30-45 minutes is long enough, after that they have a tea break which is followed by random hands (see my pre-vious column).

Initially, I produced these tents on card which needed two runs through the print-er to print both sides. How-ever, I changed the design to

paper so that my students could print their own tents for home use. The A5 sheet shown is folded first along the red line and then along the green line to make the tent. This gives the same rigidity as card. Note that the dealer’s hand needs to be upside down. One set of twenty tents can be made from printing ten pages of A4.

Each pair has a set of 20 tents. Of course, this method does need twice the number of tables needed for working in fours. Even if a pair does all 20 tents they can do them again by swap-ping hands; ie the dealer on odd numbered hands be-comes dealer on the even numbers and vice versa. You can, therefore, bid 40 hands with one set of tents.

Tents enable you to prac-tise bidding any time, any-where; all you need is a partner – some even take the tents on holiday. I have 17 different sets of tents so my students have 680 hands

to bid if they bid them both ways. Each set has a teach-ing theme so students sometimes choose to ran-domise them so that they don’t know what’s coming up next.

However, my version of Acol will be slightly differ-ent from yours; the differ-ences will be small but no-ticeable. Hence, if you like the tents idea, you should make up your own. I used MS Publisher. You can also do them in Word if your version can turn text boxes upside down.

If you would like a free PDF and/or Publisher copy of my set on basic no-trump slam bidding and the master sheet to make answer pads, email [email protected] If you have Publisher, you can delete my hands and use it as a template for your own hands. If not, you can make your own template in Word using my PDF copy as a guide – quite a lot of work, but well worth it in the long run. ■

Page 34 BRIDGE April 2016

Answers to David Huggett’s Play Quiz on page 32

NW E

S

1. ♠ A 7 5

♥ J 6 4

♦ 9 7

♣ K Q 7 3 2

♠ K Q J 9 2 ♠ 10 8 3

♥ K 9 ♥ Q 10 8 7

♦ J 8 5 4 2 ♦ Q 10 3

♣ 4 ♣ J 6 5

♠ 6 4

♥ A 5 3 2

♦ A K 6

♣ A 10 9 8

You are declarer in 3NT and West leads the ♠K. How do you plan the play?

If you think you have nine easy tricks think again, for even if clubs break 3-1 the suit is blocked if the jack is in the three-card holding.

Just duck the spade lead and the continuation. Then win the ♠A and throw a club from hand. Now the suit is unblocked. If West switches to another suit at trick two you still have the spade ace as an entry to dummy.

NW E

S

2. ♠ 9 5 4 2

♥ A 6

♦ K J 5 4

♣ 8 4 3

♠ K 7 ♠ Q J 10 8

♥ 3 2 ♥ 10 9 8 5 4

♦ 10 9 7 2 ♦ Q 8

♣ A Q 9 6 2 ♣ 10 7

♠ A 6 3

♥ K Q J 7

♦ A 6 3

♣ K J 5

You are declarer in 3NT and West leads the ♣6 with East playing the ♣10. How do you plan the play?

You know from the Rule of

Eleven that East has at least two clubs and that West pre-sumably has the missing big ones and this leads to the conclusion that it would be bad news to let East in. How-ever, with only eight tricks im-mediately available, you are searching for the ninth and this might easily come from diamonds, given a little luck. It would be a mistake to finesse against the queen, however, in case East holds that card. A safer play would be to cash the king followed by the ace before leading towards the jack. You will find your ninth trick if West holds the queen or East has the doubleton (or singleton) queen when you will have stopped East gaining the lead.

NW E

S

3. ♠ Q 6 4 2

♥ A Q 7

♦ J 9 7 5 3

♣ 2

♠ Void ♠ K 10 8

♥ J 9 8 6 5 4 ♥ 3

♦ A K 6 ♦ Q 10 8 4

♣ 10 8 7 5 ♣ J 9 6 4 3

♠ A J 9 7 5 3

♥ K 10 2

♦ 2

♣ A K Q

You are declarer in 6♠ and West leads the ♦A, ♦K. How do you plan the play?

Clearly the slam depends on not losing a trump and the odds dictate taking the fi-nesse rather than hoping for a singleton king offside. You have to be careful that East does not hold all three miss-ing trumps, so the best line is to enter dummy and lead the queen of trumps. If East cov-

ers and West shows out, you must get back to dummy for a finesse against the ten. In case of a bad heart break, it was probably right to have entered dummy the first time with a club ruff rather than a heart and the second time you can choose between either suit. Trying to enter dummy twice with hearts would not have worked.

NW E

S

4. ♠ K J 6 4

♥ J 7 3

♦ A J 2

♣ A J 2

♠ 9 5 2 ♠ 3

♥ K Q 10 4 ♥ A 8 6 5

♦ 10 9 7 ♦ Q 8 5 4

♣ 9 8 5 ♣ Q 6 4 3

♠ A Q 10 8 7

♥ 9 2

♦ K 6 3

♣ K 10 7

You are declarer in 4♠ and West leads the ♥K followed by the ♥4. East wins the ♥A and returns the ♥6. How do you plan the play?

You have a potential loser in both the minors and the con-tract might seem, superficially, to rely on some luck in not los-ing to both minor suit queens. In fact, as long as trumps are not 4-0, the contract is as-sured. Ruff the third heart high – why not? – and draw trumps. Now, simply play three round of diamonds from the top. Whoever wins the queen will be forced to either open up the club suit or give you a ruff and discard. The ♦J in dummy is a distraction, inviting you to take a finesse. Without it the endplay looks more obvious somehow. ■

Mr Bridge2016

Tutorial Weekends

Denham Grove Near Uxbridge, UB9 5DG

1-3 July £215Further Into the Auction

Will Parsons

Inn on the Prom St Annes on Sea FY8 1LU

15-17 July £199 Signals and Discards

Gwen Beattie

Wyndham Garden

Grantham Marston, Lincs NG32 2HT

21-23 October £199 Stayman and Transfers

tba

Full Board No Single

Supplement*

( 01483 489961 www.mrbridge.co.uk

*subject to availability

BRIDGE April 2016 Page 35

In theory, it may seem obvious that you are only allowed to use partner’s calls and plays in deciding what to

do, but, in practice, players find it quite hard not to use other information from partner and many players do so, quite illegally, often without realising they are doing something wrong. Let us look at a simple example. Your LHO opens 1NT, partner bids 2♣, opposition ask, and you say, ‘Both majors.’ Partner looks shocked, and blurts out, ‘Didn’t we agree to play that 2♣ shows the minors?’ You have five spades and 11 points: what do you bid after RHO doubles? If partner had just sat there quietly, no doubt you would have bid 3♠, so now you must bid 3♠, even if it turns out to be a disaster. Why? Because you are not allowed to use what partner says, and if you do not, then you have an obvious 3♠ bid.

Generally, in most clubs, rulings about unauthorised information (I shall refer to it as UI for the rest of this article) are very rare, many clubs having about one or two a year. Is this because there is not much UI being used? No, I am afraid not, use of UI is rife in clubs, but opponents very rarely call the director, and some directors would not really know what to do if they were called. Let us look at it from the point of view of the player, and in a later article from the director’s side.

All sorts of information comes to you from partner during the course of a session of bridge. Probably the most common and best known is the hesitation that shows doubt: if LHO opens 1NT, and partner thinks for some time then passes, then you know he has a problem: what should you do? Well, if you have a clear and obvious bid, pass or double, then you make it and do not worry. But if you have a borderline choice, perhaps whether to make a doubtful bid or to pass, what has partner’s hesitation told you? You know that he has points. If he only had two points he would have passed cheerfully, without hesitating. If you have a doubtful bid now, it is safer to

bid because of the hesitation, but you are not allowed to use the hesitation and you must pass.

That is an example of the basic rule: when you have UI from partner you must not just take no advantage, you must go further and make every effort possible to take no advantage. If you are going to play a legal and ethical game (and the game is much more satisfactory if played ethically), you really must bend over backwards to make sure you do not gain in any way from UI from partner.

Is this easy to do? Not necessarily. I have a highly ethical partner, but she sometimes gets confused about what is required in UI situations. Let me give you a real example: I bid 1♣, she bid 1♠, I rebid 2♣ and she decided to try for game by showing her diamonds, bidding 2♦. Unfortunately we had agreed to play the Bourke Relay, so 2♦ was artificial and game forcing, which she had forgotten. I alerted it, the opponents asked, and now she had UI. Now I bid 2NT: what should she have done? The answer is that she should think what she would have done if she had not seen my alert or heard my explanation. Of course, she might have had a choice: in that case she should have made sure that she chose the one not suggested in any way by the UI. In the case I have told you I think she should have passed 2NT since she was minimum, but she bid 3♣, trying to be ethical, and I bid 3NT. It went one off, so opponents were not damaged.

Note the problems of choice: there might be three choices you would consider, and then to be ethical you must choose the one that is least suggested by the UI. Some players say that they always make the call they would have made anyway. This is often illegal and sometimes unethical: players often consider various choices before a call and they cannot be certain when there is UI what call they would have made without it. Strangely, players who say this always choose the one most favourable for them. So

please be ethical: if you have only one obvious call then make it, but if you have a choice please always choose the one least suggested by the UI.

What other forms of UI are there? We looked at Unauthorised Panic in a recent article (BRIDGE 148) where partner describes your agreements as he believes them to be, you thought differently and your hand is not what he says! This situation and the hesitation showing doubt are the two commonest situations, but partners do other things, like sigh deeply, glare across the table, say, ‘Oh my goodness!’ after some action of yours, smile happily when you make a call or play, frown and various other things. These all give an impression of what he is thinking and so what he has in his hand or what he intends to do, so they are all UI and subject to the same laws. Be as ethical as possible, try to take no advantage, and tell partner later to behave. On occasion, you will be ruled against: so long as you have done your best to take no advantage then your actions are ethical even if a director or appeals committee decides that you have misjudged and done the wrong thing.

Suppose you feel your opponents might have UI and you are worried they will or have taken advantage, either deliberately or accidentally, what should you do? Well, first thing is to establish the UI occurred. If you felt a player has hesitated, then his partner bid, which might be affected by the hesitation, it is reasonable to say, ‘Do you agree you thought before calling?’ to the hesitator. If he agrees, then you can say you are, ‘Reserving your Rights,’ (to call the director at the end of the hand), but if he does not agree you have to call the director immediately to sort out the difference of opinion. Either way, you call (or recall) the director at the end of the hand if you are unhappy about the partner’s call. Do not accuse anyone of anything, just tell the director what happened and let him sort it out. ■

Unauthorised InformationDavid Stevenson answers your Frequently Asked Questions

Page 36 BRIDGE April 2016

Improve Your Defence with Andrew Kambites

Cover an Honour with an Honour

The saying: ‘Cover an honour with an honour’ must be seen as part of the same principle

as ‘Second Hand plays low’. This is demonstrated in Layouts A to D.

NW E

S

Layout A

♠ A J 6

♠ K 10 9 ♠ 8 7 5 4

♠ Q 3 2

In Layout A, if South leads the ♠2, West must not play the ♠K. If South leads the ♠Q West must cover with the ♠K, which promotes a spade trick for his ♠10 on the third round. Provided West is careful to use his ♠K to neutralise South’s ♠Q, declarer can never make more than two spade tricks.

In Layout A, West can see his ♠10 being promoted. Layout B is a little less obvious for West because the card being promoted is in his partner’s hand. However he should follow the same path as in A.

NW E

S

Layout B

♥ A J 6

♥ K 3 2 ♥ 10 9 8 7

♥ Q 5 4

In Layout C, again, as long as West uses his ♦K to cover either the ♦Q or ♦J he promotes a trick for his partner.

NW E

S

Layout C

♦ A 6 5

♦ K 3 2 ♦ 10 9 8 7

♦ Q J 4

From West’s perspective, Layout D looks the same as Layout C. He covers South’s honour, but that achieves nothing because it is South rather than East who has the ♣10. Still, covering doesn’t lose anything either because declarer could never be prevented from taking three club tricks.

NW E

S

Layout D

♣ A 6 5

♣ K 3 2 ♣ 9 8 7 4

♣ Q J 10

So it seems that covering an honour is good bridge. Not always! We now need to consider the exceptions.

NW E

S

Layout E

♠ A J 3 2

♠ 7 6 4 ♠ Q 5

♠ K 10 9 8

In Layout E, South leads the ♠J from dummy. East must not cover. Declarer would not be leading the ♠J unless he

had the spade intermediates (♠10 and ♠9) in his hand. He is trying to smoke out the ♠Q. Provided East plays low smoothly, declarer is quite likely to overtake with the ♠K and finesse the other way, losing to your doubleton ♠Q.

Don’t cover a jack or ten with a queen if declarer might have a two way finesse.

NW E

S

Layout F

♥ J 4 3 2

♥ Q ♥ K 7 6 5

♥ A 10 9 8

In Layout F, South opens 1♥ and becomes declarer in a heart contract. He leads the ♥J from dummy. Before covering, East must remind himself of the purpose of covering an honour, namely to promote a lesser card into a winner either in his own hand or his partner’s hand. With such poor hearts, East has no prospect of promoting winners in his own hand. Also, since South must have at least four hearts for his 1♥ opening, West cannot have more than one heart. Covering the ♥J with the ♥K cannot possibly promote winners in West’s hand, and if West has the singleton ♥Q or ♥A, East will have egg on his face!

Only cover an honour if you can see a realistic chance of promoting an intermediate card for your side.

Even if partner turns out to have a small singleton, you are making it too easy for declarer by covering the ♥J with the ♥K. Look at Layout G.

BRIDGE April 2016 Page 37

NW E

S

Layout G

♥ J 4 3 2

♥ 8 ♥ K 7 6 5

♥ A Q 10 9

If East covers the ♥J with the ♥K, declarer can easily play off the remaining three heart winners. If the ♥J wins, declarer will continue by finessing the ♥10. Now, he must waste an entry to dummy to take another heart finesse. Maybe having to use up his entries to dummy will leave him unable to take a successful finesse in another suit, or maybe he will be unable to reach dummy’s winners later in the play.

NW E

S

Layout H

♦ Q J 6 5

♦ 8 4 ♦ K 7 3 2

♦ A 10 9

In Layout H, declarer leads the ♦Q from dummy. If East covers with the ♦K, declarer will win the ♦A, cash the ♦10-9, enter dummy in another suit and cash the ♦J, making four diamond tricks. If East keeps his ♦K, the fact that East has more diamonds than South means that declarer can never make more than three diamond tricks.

NW E

S

Layout J

♣ A 10 9

♣ K 7 3 2

♣ Q led

Layout J is the same as Layout H with the cards rotated by 180 degrees. South leads the ♣Q. West has greater length than dummy so it would be unwise to cover. South is very likely to have the

♣J, and even if East has it, then the defenders make a club trick on the third round of clubs rather than the second round.

NW E

S

Layout K

♠ Q J 4

♠ 10 7 3 ♠ K 6 5

♠ A 9 8 2

Layout K is very instructive. Declarer leads the ♠Q from dummy.

If East covers with the ♠K declarer can win the ♠A and run the ♠9, finessing against West’s ♠10. He has no spade loser.

On the other hand, suppose East allows dummy’s ♠Q to hold. Declarer is powerless. If he continues with the ♠J, East covers with the ♠K, promoting West’s ♠10. If he continues with the ♠4, East plays low and declarer must play the ♠A to prevent West’s ♠10 from winning. Thus East’s ♠K wins the third round of spades.

Layouts H, J and K also illustrate another theme. It is rarely right to cover the first of touching honours.

Playing in a club of variable standard, just occasionally you will get caught by Layout L.

NW E

S

Layout L

♣ A 6 5

♣ K 8 3 2 ♣ J 10 9

♣ Q 7 4

A novice declarer leads the ♣Q, sometimes called a fairy finesse because it should never gain. (Correct play is to lead the ♣5 from dummy towards the ♣Q, hoping East has the ♣K.) An alert West will assume South has the ♣J and bearing in mind that he has greater club length than dummy, he decides not to cover with the ♣K, thus allowing the ♣Q to win. I am afraid it is an occupational hazard in defending well that sometimes a novice does something absurd and comes out smelling of roses.

In Example M, West starts with the ♦K-Q-J. Declarer wins dummy’s ♦A on the third round, following suit from his own hand, and calls for the ♠10. Should you cover with your ♠J?

NW E

S

Example M

♠ 10 6 2

♥ 6 5 3

♦ A 6 2

♣ J 6 5 4

♠ J 5 4 3

♥ Q 8 7 2

♦ 7 4 3

♣ A 9

South North

2NT1 3NT

All Pass120-22 points

You should be very wary of covering a ten with a jack when declarer might have a choice between taking a finesse and playing for the drop. Declarer has no intention of taking the finesse, especially as it would clearly be into the hand that has the thirteenth diamond.

If you cover the ♠10, declarer has an easy nine tricks. If you cherish your ♠J, declarer overtakes the ♠10 with the ♠A and subsequently tries unsuccessfully to drop the missing ♠J. South has:

♠ A K Q 9

♥ A K 4

♦ 9 8 5

♣ K Q 3

If you wanted to cover, just ask yourself what you intended to promote in partner’s hand. Declarer’s 2NT showed at least 20 points. Dummy has 5, you have 7 and partner has already shown up with the ♦K-Q-J. Partner has 0-2 points left. Declarer would hardly play this way unless he had the ♠A-K-Q. Does he have the ♠9? Ask yourself what you would do with a spade holding of ♠A-K-Q-8 opposite ♠10-6-2. The obvious play is to cash spades from the top, making 4 spade tricks if spades were 3-3 or one defender started with the ♠J doubleton. ■

Active & Passive Defence continued

Page 38 BRIDGE April 2016

The annual individual tourna-ment was into its last round. Robin Hood looked down at

the mixed results on his scorecard. Why did everyone overbid so much when they partnered him? He need-ed aces and kings to make tricks, not queens and jacks, and a partner look-ing confidently towards him. If truth be told, he would not be much above average on this occasion.

For the last three boards he was joined by Friar Tuck, Little John and Meg Jenkyns, barmaid at the Drunken Partridge for as long as anyone could remember. Hoping that he might at least have an interesting contract to play, Robin Hood sorted through his next hand. It contained no card higher than an eight!

NW E

S

Dealer West. Game All.

♠ A K 9 6 4

♥ A K 6 4 2

♦ A K 8

♣ Void

♠ Q ♠ J 10 5

♥ Q J 9 5 ♥ 10 8

♦ Q J 2 ♦ 10 9 7 4 3

♣ A K Q J 2 ♣ 10 9 5

♠ 8 7 3 2

♥ 7 3

♦ 6 5

♣ 8 7 6 4 3

West North East South

Little Meg Friar Robin

John Jenkyns Tuck Hood

1♣ Dbl Pass 1♠

2♣ 6♠ Pass Pass

Dbl All Pass

Robin Hood gritted his teeth when Meg leapt to the six-level. Did she realise that his 1♠ response had promised nothing? Could she not take the auction more slowly?

Little John doubled the slam and led the ♣A. ‘I may have overbid a bit, but you play the cards so well,’ said Meg.

I wonder where I’ve heard that before, thought Robin Hood. A dummy resembling an overflowing treasure chest appeared before him. By the Saints, the excellent Meg actually had her bid!

Robin Hood ruffed the club lead in dummy and drew one round of trumps with the ace. Since he might need to ruff three times in his hand, he abandoned the trump suit for a moment and played dummy’s two top hearts followed by a low heart.

Friar Tuck, who held a natural trump trick with his ♠J-10, discarded a club. It could hardly gain to ruff with a trump winner. For all he knew, Little John was about to win this trick with the ♥Q. Hood ruffed in his hand, returned to dummy with the ♦A and ruffed another heart, establishing the thirteenth card in the suit. A diamond to the king permitted a diamond ruff in his hand. He returned to dummy with a second club ruff.

Dummy now held three trumps and a good heart. Friar Tuck eventually claimed his trump trick but the slam was made.

‘Why did you double, John?’ protested Friar Tuck. ‘You didn’t make a single trick.’

Little John’s eyes bulged. ‘Are you sitting there telling me you wouldn’t double a slam when you held 18

points?’ he exclaimed. ‘If you are, I don’t believe you.’

‘Don’t believe me then,’ persisted Friar Tuck. ‘If they were too high, we’d get a top whether you doubled or not.’

‘The highest card in my hand was an eight,’ said Robin Hood.

‘Yes, but you play the hands so beautifully,’ declared Meg Jenkyns. ‘Players like you don’t need high cards.’

The players changed seats, leaving Robin Hood in partnership with Friar Tuck.

NW E

S

Dealer East. E/W Game.

♠ J 2

♥ 7 6 5 3

♦ 7 6 4

♣ A K 6 4

♠ A 8 7 6 4 ♠ Void

♥ K Q J ♥ A 10 9 8 4 2

♦ J 8 ♦ 10 9 5 3

♣ J 8 2 ♣ Q 10 7

♠ K Q 10 9 5 3

♥ Void

♦ A K Q 2

♣ 9 5 3

West North East South

Friar Meg Robin Little

Tuck Jenkyns Hood John

2♥ 4♠

Dbl All Pass

Friar Tuck doubled the spade game and led the ♥K. ‘You lucky er... rascal!’ he exclaimed when the comely Meg laid down the ♣A-K in the dummy.

‘Could be worse, it’s true,’ replied Little John.

Robin Hood maintained a

Robin Hood’s Bridge Adventures by David Bird

Robin Hood’s

Awful Hand

BRIDGE April 2016 Page 39

cheerful disposition but he could guess that dummy’s ♠J might be a useful card too. Had Friar Tuck not learnt a lesson on penalty doubles from the previous board?

Little John discarded a club on the first trick and ruffed the heart continuation. A club to the king was followed by a second heart ruff in the South hand. A club to the ace and a diamond to the king and ace left these cards still to be played:

NW E

S

♠ J 2

♥ 7

♦ 7

♣ 6 4

♠ A 8 7 6 4 ♠ —

♥ — ♥ 10 9 4

♦ — ♦ 10 9

♣ J ♣ Q

♠ K Q 10 9

♥ —

♦ Q 2

♣ —

Little John considered his next play carefully. If he led the ♦Q, Friar Tuck might ruff and continue with ace and another trump. Declarer would then have to concede a diamond trick for one down. What would happen if he led the ♦2 instead at this stage?

It seemed that he would make the contract whether West’s last plain-suit card was a diamond or a club.

Robin Hood won the ♦2 with the ♦9 and had no trump to play. Little John ruffed the heart return with the nine, West overruffing with the ♠A and returning a trump. Little John won in his hand and ruffed the ♦Q with the ♠J. The contract had been made.

‘Don’t say it if you know what’s good for you!’ proclaimed Friar Tuck, glaring across the table.

‘Say what?’ asked Robin Hood.‘That I shouldn’t have doubled,’

the Friar continued. ‘With five good trumps I’m doubling – so is any man with red blood in his veins. I’m not going to wait until I hold six trumps.’

Meg Jenkyns entered a second

Robin Hood continued great board on her scorecard. What an excellent round it had been so far. Perhaps she could make it three good boards in a row.

NW E

S

Dealer South. N/S Game.

♠ 8 5

♥ A Q 10 6

♦ A 6 4

♣ J 9 8 5

♠ Void ♠ Q J 10 9 7

♥ K 9 3 2 ♥ J 4

♦ Q 10 9 2 ♦ J 5

♣ A K Q 4 3 ♣ 10 7 6 2

♠ A K 6 4 3 2

♥ 8 7 5

♦ K 8 7 3

♣ Void

West North East South

Robin Meg Little Friar

Hood Jenkyns John Tuck

1♠

Dbl Rdbl 2♣ 2♠

Pass 3♠ Pass 4♠

Pass Pass Dbl All Pass

When Friar Tuck edged upwards into 4♠, Little John thumbed through his cards. If ever anyone had a sound penalty double it was surely now. He held three certain trump tricks in his hand and partner had shown a good

hand too. ‘Double!’ he said.Friar Tuck ruffed the club lead

and cashed the ♠A, not in the least surprised when West discarded a club.

What could be done now?Friar Tuck aimed to score as many of

his low trumps as possible. He finessed the ♥Q successfully and ruffed a club in his hand. After playing the king and ace of diamonds, he ruffed a third club. A heart to the ace allowed yet another club ruff and the trump king was then his tenth trick. Robin Hood was left staring at three side-suit winners, while Little John still held his ♠Q-J-10.

‘I was lucky to find the entries to take so many club ruffs,’ observed Friar Tuck. ‘Any lead but a club, Robin, and I can’t make it.’

‘You see!’ cried Little John. ‘Nothing wrong with my double. You gave it to them with your stupid opening lead.’

‘Let’s not fight about it,’ replied Robin Hood. ‘Many players before me have made the mistake of leading from an ace-king-queen combination.’

‘Just look at the scores I’ve collected on this round!’ exclaimed Meg Jen-kyns. ‘Plus 1660, plus 590 and plus 790.’

‘You played excellently, Meg,’ said Robin Hood, rising to his feet and giving her a brief hug. ‘You’ve more chance of winning the big prize than I have. That’s for sure!’ ■

Page 40 BRIDGE April 2016

How Things Were

Bridge boomed in the 1960s and 1970s. At that time, many young players came into bridge, by and large untutored – but that did not seem to matter. Those who wanted to improve did so by reading, by peer group conversation, but above all by playing. Most clubs were cut-in rubber bridge clubs and if you joined a club you learnt as you played. In cut-in bridge you played with anybody and everybody in rotation, so systems were on the whole very simple and, within any one club, very convergent. System discussion more or less started and ended with an agreement to play strong or weak no-trump and possibly a defence to pre-emptive bidding.

This period also co-incided with the rise of duplicate bridge. Duplicate bridge offers a more rigorous and competitive form of club bridge which proved attractive to the younger players. Rubber bridge clubs had been adversely affected by the small print of the Betting & Gaming Act 1960. The English Bridge Union, founded as a successor body to the Duplicate Bridge Control Board, distanced itself from rubber bridge and began to promote itself as the National Governing Body for duplicate bridge and duplicate bridge only.

Many of today’s higher level players date their involvement with bridge from that time. They were nearly all self-taught. It is rare to find players from that era who actually had formal lessons in contrast to learning on the job. As duplicate bridge spread, it

English Bridge Education & Development (EBED), the charity arm of the EBU, is proposing a new learning channel for bridge, involving a much swifter transition from learner to player. As an

interested outsider with a 10-year plus background in teaching in this style, I feel qualified to comment.

unleashed a substantial improvement in bidding systems and a proliferation of conventions. This extended the apparently needed body of knowledge to become an entry level bridge player.

The Classroom Approach to Bridge Fundamentals

The 1970s was also a period of well-funded adult education. Courses were available not only in academic, professional or vocational subjects but in leisure time activities as well. Bridge with its burgeoning popularity and its apparent complexity lent itself well to this format of teaching. Local authorities recruited bridge teachers who quickly became used to teaching bridge within academic year timetables. Bridge always offers something new – there is always more system to practice, new conventions to learn, and instructional deals to play. Bridge teachers had no difficulty filling their classes or their timetables.

During this period, many bridge clubs changed in character and be-came duplicate only. While sessions were offered at different levels, novices were not generally welcome in weekly duplicates, but were directed towards continuing formal classes as a way of gaining their first years of experience. Funding for adult education eventu-ally became more difficult but larger bridge clubs took over the role of local authority classes.

The Outcome For Clubs

Over the last twenty years or so, bridge

has worked almost exclusively to this model. This is good – up to a point! Those who have learnt this way tend to be those who have benefited most from extended classes. The classroom system plays to the strengths of those who like to learn and stick to ‘rules’ for their bridge. Playing this way, with a system largely learnt by rote, has its plus points. It is far better to play like this than to be a maverick or flair player who can be almost impossible to partner. However, it is a fact but those who stick to ‘rules’ and cannot see a little further, tend to have a lower ceiling to their ultimate bridge ability.

The classroom route therefore tends to produce players who settle at middle to low club level, happy to stick to club and social bridge. Those unattracted by, bored by or unsuited to the extended class route, often get lost to bridge completely. One of the results has been fewer players coming through to the higher echelons of bridge. The extended classroom system is also poorly adapted to younger learners. Those younger people who are getting into bridge have by and large ‘beaten the system’ by learning at school, college, or from friends or family, not at clubs. There is a significant group of younger players who play at county or national congresses at a good standard. A common theme is that these players quickly show a good understanding of card play technique, and also understand how bidding systems work, rather than learning a single system by rote. This enables them to discuss and espouse different bidding systems and acquire

Accelerated Bridge Teaching

From Learner to Player in Three

Months or Fewer by Ned Paul

BRIDGE April 2016 Page 41

their own taste in bidding. How many young players at national level are playing basic Standard English? I would suggest none.

Accelerated Bridge Teaching

If a programme of Fast Track or Accelerated Bridge Teaching could be successfully introduced, it would give bridge a chance to attract and retain a larger pool of both younger and higher potential ceiling players. Running alongside the traditional model, fast track learning would give potential players a choice of learning channels, and most players would self-select the option best suited to their interest, needs, ability and available time.

The number of players who might benefit from Accelerated Bridge Teaching (ABT) is likely to be smaller than those who are suited to traditional extended teaching. This is one reason why the traditional model has persisted. It suits clubs to have settled classes with viable numbers; it suits them organisationally and it suits their cash flow. However, it is of dubious benefit to the real long-term future of bridge, if the brighter people, the potential movers and shakers of bridge in the future, get overlooked or turned off by the only learning system on offer.

What specifically do we mean by Accelerated Bridge Teaching? A fast track programme should concentrate on teaching the basic mechanics of bridge in as short a time as possible and then add a simple but modern bidding system. After this, learners should be encouraged to play and play and of course there are plenty of resources out there, both printed and online, for them to be able to study as well as play. Clubs could be pleasantly surprised at how quickly bright learners could be integrated into club games, but it would need both a willingness by the clubs to allow new players into the games and a willingness by better players to mentor and even partner the ‘coming’ players.

Fast Track Bridge

The projected ABT timetable suggests about three months of initial lessons. What would I try and teach in this time and what system would I concentrate on? The first thing to teach is an understanding of how bidding systems work. A bidding system is an attempt to communicate your hand to partner and to combine to best effect you and your partner’s resources. It’s a team thing not a solo effort. The questions you should be asking constantly are ‘What have I shown my partner?’ ‘What has my partner shown me?’ And ‘How do these two hands combine and where are we going?’

A bidding system involves choices to cover all the available hand patterns and these choices can be simpler or more complex, better or worse. Bad choices tend to be ‘Darwin-ed’ out of existence: eg who plays 3NT for take out of a pre-empt these days, or who plays Swiss responses to one-of-a-major openings? But conventions like

Gerber live on because it is simple to learn and its disadvantages are subtle. The text book for an ABT programme could well be entitled ‘Understanding Bridge’ and would have many of the elements of an updated version of the S J Simon classic Design for Bidding.

Lesson Specifics – What To Teach

England is Acol-land so I would definitely use an Acol 4-card major base. I would also start with a weak no-trump. This is an incredibly easy system to learn the basics of and is effective at beginner level. The weak no-trump is especially effective at duplicate pairs. The system can also provide a good base to start varying into other ‘natural’ systems. It is quite easy for weak no-trump players to transition into a strong no-trump, but practice shows it is quite hard for strong no-trump players to really understand the weak no-trump. The same with 5-card majors: you can’t

Accelerated Bridge Teaching

The Effectiveness of Transfers

NW E

S

Dealer North. E/W Vul.

♠ A 10 8

♥ Q 7 4

♦ A 6 5 2

♣ K 9 5

♠ 7 5 2 ♠ K 6

♥ J 6 2 ♥ 10 5 3

♦ Q 7 4 3 ♦ 10 9 8

♣ A J 6 ♣ Q 10 8 7 4

♠ Q J 9 4 3

♥ A K 9 8

♦ K J

♣ 3 2

West North East South

1NT Pass 2♥

Pass 2♠ Pass 3♥

Pass 3♠ Pass 4♠

All Pass

Using transfers in response to no-trump bids allows the responder two chances to describe their hand before a decision about game needs to be made. The transfer

itself shows a 5-card suit and when partner ‘completes’ the transfer, the responder gets the opportunity to further describe their hand by passing, inviting or forcing to game.

With game values and 5-4 in the majors South starts by bidding 2♥, showing five spades. When North completes the transfer, South then bids 3♥ showing four hearts as well. This new suit after a transfer is forcing. The bidding should then advance to 4♠.

Using traditional, non-transfer methods South can use Stayman and ignore the fifth spade, or invite game in spades and ignore the 4-card heart suit. Yes, there are agreements you can make to get round this – but they are more complicated than transfers! My thesis is why not teach transfers from the outset?

There are no problems in the above deal in playing 4♠ – declarer loses just a spade and a club.

3NT should not make on a club lead.

Page 42 BRIDGE April 2016

play 5-card majors without playing negative doubles (to overcome in-tervention) so it’s easy to start with ‘longest-suit-first, major-before-minor’ Acol and then add from there.

The brand of Acol taught should be modern tournament-adapted Acol. The programme must include teaching some of the modern bidding choices from the outset. Current bridge teaching is geared to teaching players social bridge as played in the 1970s. To progress to effective club level, current learners who have started with Standard English, have to unlearn and discard a lot of stuff that should not have been taught in the first place.

Take, for example, responding to a 1NT opening bid in an unopposed auction. Stayman and transfer re-sponses to 1NT should be taught as standard. These particular conven-tions are not complicated and are re-garded as standard by most club play-ers. Just teach people that’s what you do, then as beginners they accept it and espouse it. Incidentally, the full rationale for transfers is widely mis-understood and misexplained within the teaching community. Nearly all you hear is that, ‘the strong hand be-comes declarer.’ This is only true when responder has a weak hand. The true value of transfers is that responder

Accelerated Bridge Teaching makes a bid and then through the transfer mechanism effectively retains the bidding to make a further descrip-tion of their hand. At this second stage responder can show weakness (through passing), invitational values, strength, two-suited shape, etc. Trans-fers effectively double the number of bids responder has, with a resultant huge increase in the precision of the responses. Consequently more accu-rate final contracts are reached.

The same applies to 1-of-a-suit openings. Limit bids live on, but other modern ideas should be included alongside and taught from the outset: the ‘weak freak’ raise to game, splinter bids, game-forcing 2NT are all much better weapons than ‘traditional’ methods so why teach old stuff that has to be replaced?

Opening two bids the same. Acol strong twos are amongst the most widely misunderstood bids with club bridge players. Their effectiveness has also really passed. When teaching two bids, I give my classes a range of strong and weak two bid candidate hands. Given a choice and an understanding of how the bids work, they always voluntarily choose weak twos over strong twos. If strong twos are the past, weak twos are the present, but we live in times when bidding is developing rapidly and portmanteau (‘multi’) two bids are very likely the future. Standard weak two bids are fine as a beginning though – learn to walk

before you run. Curiously, if players are going to play mainly social bridge with their chums and not club bridge, then they do need to understand and be able to play strong two bids as well. So again, what should be taught is an understanding of the bids and what you are trying to achieve, not just the ‘rules’ for making the bids.

Defensive bidding. Overcalls and take out doubles are usually well taught but unassuming cue bids in competition again should also be taught early. When partner opens, say 1♥ and opponents overcall 2♣, there are two reasons to call 3♥ – offensive and defensive. This should be an easy concept to grasp. So, direct raises are pre-emptive level-of-the-fit type bids and defensive-minded, and the UCB is value-showing, triggering the offensive channel.

Summary

These are just some examples of how, if you explain the concepts properly, the brightest beginners can easily adopt supposedly advanced bidding methods from the outset.

Of course, some players who find their way into the ABT channel, will find some of these ideas challenging, but at an ABT system should be at least ‘bright lite’ and not just traditional social basic Standard English.

The biggest issue is perhaps that some of the more ambitious stuff is outside traditional teachers’ comfort zone, and in some cases outside their experience. Teacher workshops, where the issues can be discussed, may have a role here.

Just at present, bridge is receiving quite a bit of media attention and there are early indications that bridge could be coming into an upturn of interest among non-players.

A fast track programme of bridge teaching would be an ideal vehicle to capitalise on such interest and to capture the greatest possible number of higher-ceiling players for the po-tential good of the game. There will be challenges along the way though, both within and without bridge, to creating, installing, supporting and succeeding with an ABT pro-gramme. ■

Weak Twos in Action

NW E

S

Dealer East. N/S Game.

♠ 4 2

♥ A K 10 4

♦ K 10 8 5 2

♣ K 3

♠ A 9 8 3 ♠ K Q 10 7 6 5

♥ 7 ♥ 8 6 3

♦ Q 6 3 ♦ 9

♣ A 10 8 7 2 ♣ J 9 4

♠ J

♥ Q J 9 5 2

♦ A J 7 4

♣ Q 6 5

West North East South

2♠ Pass

4♠ All Pass

East has a good weak two bid and South has just too little to double. West realises that the pre-empt must be extended and raises directly to 4♠, the level of the fit. This effectively stops North entering the auction, yet given a clear run North-South would most likely buy the contract in a heart contract. 4♠ makes with the aid of a double finesse in clubs but North-South can make up to eleven tricks in hearts.

BRIDGE April 2016 Page 43

It was a fairly casual and relaxed at-mosphere at the Riverside as mem-bers gathered for a mixed pairs

competition. Mingling cheerfully in the bar before the bridge, most of the familiar faces were present. ‘I see Mar-jorie’s here today,’ observed Millie to no one in particular. She whispered in my ear, ‘She’s so pedantic about pro-nunciation, she can hear an H dropped two tables away.’ Approaching the start time the TD cajoled us into the bridge room. This board caused much discussion as it progressed round the room.

NW E

S

Dealer North. N/S Vul.

♠ A 4 3

♥ K J 7

♦ A K

♣ A K Q 10 9

♠ Q 8 6 5 2 ♠ K J 10

♥ 10 5 4 2 ♥ 8 6

♦ 2 ♦ J 6 4 3

♣ J 4 3 ♣ 8 7 6 2

♠ 9 7

♥ A Q 9 3

♦ Q 10 9 8 7 5

♣ 5

Playing with Spouse, we reached the small slam in no-trumps declared by me as South. Marjorie as West, showing no signs of language affectation, led the spade five and when dummy tabled his hand, it first looked as if Spouse and I had missed the grand slam. However when I tested diamonds by playing the ace and the king, I found East with four to the jack. With spades wide open, it became evident that an alternative strategy was required. I turned to plan B, and played the four winning hearts and the diamond queen. The jack of

clubs was the next problem. With seven clubs still out, my options were finesse and hope, or run the suit and hope. I paused to consider which to try. ‘Where am I?’ I asked. ‘Playing bridge,’ responded Spouse inanely. ‘You clot, hand or dummy?’ I demanded. He confirmed I was in hand and I led the club to the ace. I continued clubs from the top. The jack dropped and thirteen tricks arrived for six no trumps plus one, with five club tricks, four hearts, three diamonds and the ace of spades. As it happened the club finesse would also have worked; we had indeed missed the grand. ‘I’m getting plenty of practice as declarer today,’ I remarked. ‘You need it,’ was his reply, unworthy of partner and spouse. I chastised him for the comment, but of course he was only joking. ‘I’ve been dummy a lot this session. It’s like watching paint dry,’ he pleaded in self-defence.

The kind club break meant that grand slams were also making in clubs (5-1 fit) and in hearts (4-3 fit). It was not a major surprise to find that no pair had bid either of them. The traveller indicated that several partnerships had reached six diamonds, losing one trick to the trump jack, thereby making their contract on the nose.

One pair, Millie and Justin, bid the grand slam in diamonds playing against Kate. As West, Kate’s partner led the five of spades, Millie as declarer won with the ace, tested diamonds by playing the ace and the king and got the bad news about the break. So she now switched to a trump reduction strategy to cut the number of trumps in her hand, to equal those in Kate’s sitting East. She played two top clubs, ditching a spade on the second one. Then she ruffed a spade and returned to dummy with a heart. Another spade was ruffed and again she returned

to dummy with a heart. This was the position after winning the first nine tricks.

NW E

S

♠ —

♥ 7

♦ —

♣ Q 10 9

♠ Q ♠ —

♥ 10 5 ♥ —

♦ — ♦ J 6

♣ J ♣ 8 7

♠ —

♥ A Q

♦ Q 10

♣ —

Millie led out the club queen from dummy and flamboyantly discarded the ace of hearts with a spectacularly grand gesture. Then leading out the club ten she discarded the queen of hearts in a similar elaborate manner. Finally, the last club was played to trap the jack of trumps and bring home the grand. Unnecessarily, she celebrated football style by raising both fists above her head, and was promptly shown a yellow card by the TD for exuberant behaviour. At least she refrained from the spectacle of sliding on her knees across the floor. This was a deal in which thirteen tricks were available in clubs, diamonds, hearts and no-trumps. Celebrating at our local hostelry later with a round of drinks, we congratulated Millie on her declarer play. Needless to say, she accepted the praise with her usual restrained aplomb declaring, ‘If you mess with the best, you go down with the rest.’ We all stared at her and made no comment as she offered high fives around the table. She clearly watches too much reality television. ■

The Diaries of Wendy WensumEpisode 48:

A Grand Gesture

Page 44 BRIDGE April 2016

MondayI have a lot on this week – oodles of Bradt work and a fair amount of other stuff too. I start the week, as has become normal, with an online session of bidding practice with Allison. Then I set off to meet Canadian friend Katie Thorpe who is over for a few days. She wanted to go to Bletchley Park, and as I had never been and it is not so far away from here, that is what we decided to do. I meet her at Bletchley railway station and then we go on to Bletchley Park itself. It really is a fascinating place and we wander around happily, alternating stops for refreshments and gossip with having a good look at all the wartime settings, and the amazing code breaking regime.

I am home by late afternoon and get on with work. I am expecting Brian to stay the night after playing bridge in London, so I stay up late waiting for him. Eventually, around 1am, I email him to see if he is hopelessly lost (it isn’t an easy place to find, particularly in the dark). He replies that he was so hopelessly lost he checked into a hotel.

TuesdayAfter a lie-in after my late night, I drive to Aylesbury via Stoke Mandeville (Briony had forgotten her lunch). I have a meeting there to discuss some possible future work. In the evening after an hour’s coaching session, I settle down to more work.

WednesdayThere has been some sort of landslide affecting the trains between Hemel Hempstead and London, so I decide to go to London on the tube from Chesham. It turns out to be a pretty efficient service and good for going to

the Acol Bridge Club. I’m playing with Allison. I like the Wednesday daytime duplicates – for just £5 extra you get an excellent lunch. Afterwards, I go back to Shepherds Bush for my online session with Richard and Gerry.

Then it is a Super League match at the Young Chelsea. This week we’re up against Barry’s team (Hinden), but he isn’t playing. It is a match of two halves. The first half is really flat. After 12 boards we are losing by 2 IMPs (we have 3 and they have 5). In the second half our opponents bid and make slam on three of the first four boards, and then we bid and make 6NT on the next.

They do rather well on the first board, and it costs us a slam swing:

NW E

S

Dealer South. Game All.

♠ 5

♥ A 8 3

♦ A 7 5 3 2

♣ K 5 4 2

♠ Q J 9 8 4 ♠ K 6 3

♥ K Q J 5 4 ♥ 10 9 7 2

♦ 10 9 ♦ K J 4

♣ 3 ♣ J 10 9

♠ A 10 7 2

♥ 6

♦ Q 8 6

♣ A Q 8 7 6

West North East South

1♣

2♣1 2♥ 4♥ 5♣

Pass 5♥ Pass 6♣

All Pass 1Majors

North-South are not a regular partnership and North improvises somewhat with his 2♥ bid, which is intended and interpreted as a decent hand with the minors. So when East

bids 4♥, South is very suitable and bids 5♣ – now North’s hand is huge and he makes a grand slam try on his way to 6♣. It’s not the best slam in the world, but is improved by the bidding and, with the cards as they lie, declarer can hardly go wrong.

A few more IMPs drift out and we eventually lose by 10 IMPs. Then after a quick drink I get a lift home with Ed.

ThursdayI had been going to spend the day with my old Nottingham friend Carol, but she cancelled a couple of days ago, which is just as well as I am inundat-ed with work. I am up at 7.30am and work right through to bedtime. First doing final proofs of the Peloponnese, and then first proofs of Mauritius.

Also, today we pick up The Big Bridge Quiz from the printers – and it looks great. This is a fundraising enterprise to raise money for Briony’s charity: Stoke Mandeville Spinal Research. It costs £10 (all of which goes to the charity) to enter, then you can take your time before submitting your entry by the end of April in order to have a shot at winning the £1,000 first prize. To get your copy, please email Briony ([email protected]).

FridayToday is the start of the Senior Cam-rose Trials in Solihull. I am playing with Nicola, and Barry with Robert Sheehan. We start well with a win by 46 IMPs against Chris Dixon’s team. This is my favourite deal (see next page). They do say that the one contract you should never play in is 5NT. Nicola leads a heart which declarer wins in hand with the queen. He follows this by running the nine of clubs.

Seven Days by Sally Brock

BRIDGE April 2016 Page 45

Seven Days continued

NW E

S

Dealer East. N/S Vul.

♠ K J 8

♥ K J

♦ K 9 8 6

♣ K J 10 3

♠ Q 6 5 2 ♠ 10 7

♥ 6 ♥ 10 9 8 7 5 4 2

♦ Q 7 5 4 3 2 ♦ J

♣ 5 4 ♣ A Q 6

♠ A 9 4 3

♥ A Q 3

♦ A 10

♣ 9 8 7 2

West North East South

3♥ 3NT

Pass 5NT All Pass

I win with the ace! And play another heart. He wins in dummy, plays a spade to his ace, a spade to the jack and the king of spades. Then he plays a diamond to his ace and, hoping to exert pressure on West, cashes the ace of hearts before playing a club to the jack and my queen. I have three more hearts to cash so that is three down.

SaturdayToday we start against the Irens team (Forrester, Kendrick, Selway, Simpson). The first half is dire and we are 37 IMPs down. However, we bounce back in the second stanza to gain 44 and eventually win by 7 IMPs. It is a similar story in the second match against the Manchester team captained by John Sansom. A pretty bad first set in our room leads to a deficit of 16, and in the second half we gain 17 for a tiny win by 1 IMP. We are undefeated at least – not that that counts for much. At the end of the day we are in second place, just a few VPs behind Irens. Teammates have been playing brilliantly and are heading the Butler, while we wallow in the middle.

In the evening, we go out for a Thai meal – it’s nice to finish the bridge by 7.30pm so we can go out for a leisurely dinner and enjoy a glass of wine.

SundayWe start against Mossop, one of the best teams in the competition. The first half goes swimmingly and, despite losing 11 IMPs for bidding a poor, failing slam on the last board, we are 35 IMPs up. The second half is flatter and we add another 5 IMPs to our tally. However, the really good news from our point of view is that the Irens team loses to Dixon’s.

The position going into the final match is that if we win by 6 IMPs we cannot be caught. As luck would have it that is precisely how many we are up at half-time, so it is quite exciting, and the Irens team are piling on the IMPs against last-placed Sansom. But all is well. This is our main gain in the final set:

NW E

S

Dealer East. N/S Vul.

♠ J 7 5

♥ K 10 5 3

♦ K 7

♣ A 8 5 3

♠ K Q 10 9 8 6 4 ♠ 3

♥ 9 6 ♥ 8 4

♦ J 9 3 ♦ A Q 6 5 4 2

♣ J ♣ Q 7 6 2

♠ A 2

♥ A Q J 7 2

♦ 10 8

♣ K 10 9 4

In the other room, Robert is left to declare 4♥, a contract he makes on the nose. This is the bidding at our table:

West North East South

3♦ 3♥

4♠ 5♥ Dbl All Pass

The combination of the preempt and the jump to 4♠ makes life very difficult for North. When he presses on to the five level, the board is lost for him. My double and his partner’s misguess in the play leads to a 500 penalty and 15 IMPs for us.

After the bridge I put my teammates on the train for London before having a pretty miserable drive home in the pouring rain. Briony has my dinner ready and I am more than ready to collapse in front of the TV. ■

Southampton • Falmouth, England • Cork (from Cobh), Ireland

• Dublin, Ireland • Southampton

Irish Christmas Markets

Great value Mr Bridge fares from:

Inside twin cabins £479pp

Outside twin cabins £529pp

Balcony cabins £759pp

Suites £859pp

Twin outside cabin for sole occupancy £629pp

Terms and conditions apply – see Fred. Olsen 2016/17

worldwide brochure.

For reservations call Mr Bridge

on 01483 489961www.mrbridge.co.uk

Daily bridge on board, bridge fees included. Mr Bridge welcome & farewell drinks parties. Partners for single players.

No single supplements on selected twin grades of cabin.

Sails from Southampton 14th December 2016

5 nights • Balmoral • L1629

Balmoral

FOC322363_MrBridge_1/3_page_ads_W.C.18.01.16_V4.indd 125/01/2016 13:59

Page 46 BRIDGE April 2016

READERS’LETTERSBAD TO WORSEIn the Guardian’s Section 2 recently, there was a feature deploring the epidemic of cheating, not only in most games at the highest level, but also in bridge’s top competitions. It went on to report that an investigation is taking place. I was shocked. If this is so, could we please have full coverage?

I am old enough to vaguely remember the allegations concerning the Boris Shapiro – Terence Reese affair. The partners, it was claimed, disclosed heart holdings by displaying numbers of knuckles. The story goes that Reese was so amused at such a laughable gibe from sore losers, that he published Story of an Accusation, analysing every hand with heart holdings.

From this, it emerged that the partnership fares slightly worse from them than their other hands.Mr Harold Trace,Ashford, Middlesex.

SHORT HAULI have been on many of your holidays and weekends, but because you don’t have a land-based holiday in January or February, I have had to go away with another company.

I have been writing on my end of holiday comment forms, faithfully completing them after all sorts of events over the years, that I would like to go somewhere in the winter that is not long haul. Can you possibly set something up for your

devoted followers that fits the bill? The Canary Islands for instance? I had decided to go to Croatia with you in September, but am informed that the holiday has been cancelled. I am now considering a cruise but I prefer land based holidays.Mrs Mary Ward by email.

AS OTHERS SEE USYou should update the mag. Too upper class and ageist.Name & Address supplied.

DEALINGThe article Humans Can’t Shuffle, see BRIDGE 159 page 42, contains interesting observations. There is, however, one point I query, regarding the number of touching cards in a truly random pack. I would welcome comment on the following which suggests this number is greater than the zero as stated in the article.

In a truly random pack, the card following any particular card is truly random from the remaining fifty one cards. Therefore the probability that this card will be the touching card is exactly one in 51. So for the complete pack the probability of touching cards is significantly greater than zero.

This does not prevent the design of special packs with zero touching cards. Many such possibilities exist, perhaps the simplest is where each card is followed by a card of a different suit. Mr John Douce, Coventry.Ian Dalziel replies:John Douce is quite correct.

A random deal could and indeed should contain touching cards. I didn’t intend to imply otherwise.

However, a shuffle can only start to be random when every card has been separated from its original neighbours. My ‘ordered pack’ with every card touching was just a means of testing this. It is perfectly true that ‘touchers’ might ‘meet again’, and if so, that’s fine. In my diagram, no two cards had been separated and met again in one minute of shuffling, if they had, they wouldn’t count as touchers for the experiment. With continued shuffling there would certainly be some touching cards. This test can’t check a riffle shuffle – for one accurate riffle shuffle will kill all touching cards, but that isn’t random.

ANSWER PLEASEIn his biography on the EBU website, Andrew Kambites comments that most players would be better off playing no conventions, but to depend on having an extensive system instead.

Would you please ask him to expand on these views in a future article?John Martin,Dingwall, Ross-shire.As this is a frequently asked question I will ask Andrew Kambites to explain.

Watch these pages.

SHUFFLINGI refer to Ian Dalziel’s article on page 42 of BRIDGE 159. I am a competent riffle shuffler and in a series of trials typically find only two pairs of adjacent cards remain after three or four shuffles (about 4-5 seconds per shuffle), and it seems to be a different pair after each set of shuffles.

The method I use to ensure a good outcome is

to deal five piles of cards and, after the fifty-second card is dealt, lift the fifth pile and continue dealing at hand three. This achieves four hands of 13 cards and, I believe, conforms to the rules. Self-evidently, if you start with a ‘sorted’ deck, this deal will ensure there are no touching pairs. In practice, we find that the method generally yields an interesting variety of hands. Mr Bill Aitken,Edinburgh.This is just what I do and the method we recom-mend to our bridge hosts.

THE STONG NO-TRUMPAt last, what a delight it is to have one of your articles, this one by Jeremy Dhondy, devoted to explaining the attributes of the strong no-trump and five card majors. I hope you will continue to increase the number of articles covering the benefits of the strong no-trump. It should make your magazine more balanced and attractive to a wider readership. Wg Cdr Patrick Cliff, Uckfield, East Sussex.

WELL DONEI have been a reader of the magazine since I started playing bridge around 12 years ago.

In that time, I have always found the magazine to be informative and containing a variety that offers something for everyone.

I believe that the quality has gone up another notch with the addition of Teachers’ Corner giving another and practical view of bridge.

You can never please all of the people all of the time, but you come mighty close.

Keep up the good work.Mr Reg Hull by email.

BRIDGE April 2016 Page 47

SOLVED AT LASTNo questions. But I am rather pleased with myself that at the age of 93, I have ‘cracked’ the Pot Boiler. Obviously those bridge weekends under Bernard’s tutelage have paid off!Mrs Betty Tompson,Sandy, Beds.

I’LL HAVE A LOOKIn December 2014, a correspondent suggested putting a Bernard Magee convention card in the online library. Is it there yet?Mr J Jeffrey,Portsmouth, Hants.

TORCH PENHome again after a wonderful happy cruise in spite of the various hiccups with the itinerary and the shore excursions. A lovely, friendly bridge party hosted by a super Mr Bridge team.

I thought you would be interested and very pleased to know your pen/torch really came in to its own last night.

I had put it in the passport holder for the journey home and as I walked in through my front door, the electricity went off. It had been on as I arrived in my taxi as I have security lights. No problem thanks to the torch/pen! Turned off the alarm, left the cases in the kitchen, checked with my neighbours who told me it had gone off earlier as well, and went straight up to bed accompanied by my new wonderful, useful pen.

Thank you again – such a clever idea.Dorothy Pickering by email.

VOLUNTEER SOUGHTI am a committee member of Farnborough U3A. We do not have a member who

is willing to teach bridge to a beginners’ group. Do you know of anyone who might be able to help us?Mr Rob Lane by email.

SANDOWN, IOWI have just returned with three friends from a most enjoyable bridge event on the Isle of Wight with Bernard Magee and his team.

Knowing the island well and having had two Yarboroughs during the course of the weekend, I took my three companions to Culver Cliff en route to the ferry where there is a monument to the Earl of Yarborough, after whose son an honour-free hand is so named. That’s right. A hand with no aces, kings, queens, jacks or tens.

It was our first Mr Bridge experience. The venue and course content was excellent. We will definitely return.Margaret Thornton by email.

CLUB PLUGHart Bridge Club, having just moved to a larger venue, can now accommodate more players on Tuesday evenings. We are now at Fleet Methodist Church. Contact Tony Baker on ( 01635 868337 or by mobile on ( 07984 626118.Tony Baker by email.

CONTACT LISTWe would like to suggest that you introduce some kind of contact page through which people can meet others in their area for a game, along the lines of your charity bridge column.

There seem to be a lack of bridge players in the Midlands. Since we moved here, our bridge is limited.Colin & Helena Gillby email. ■

Write to Mr Bridge at: Ryden Grange, Knaphill, Surrey GU21 2TH or e-mail: [email protected]

MAY 2016

6 STAMFORD BURGHLEY ROTARY CLUB Tinwell Village Hall, PE9 3UD. 1.15 for 1.30pm. Bridge Tea. £8 pp. Rtn Alan Kinch ( 01780 444276 [email protected]

13 TWYFORD SINGERS Charvil Village Hall, Charvil, RG10 9TR. 1.30 for 2pm. 5th Annual Bridge Drive. Tea, raffle. £32 per table. Gillian Matthews ( 01189 479986

15 CHIGWELL RIDING TRUST for people with special needs. Abridge Village Hall, Essex. 1.30 for 2pm start. £12 pp. Duplicate/Chicago available. Mrs M Walker ( 02035 390387

20 CHILDREN’S CHARITIES. Doddington Village Hall, March. £15 pp. Val Topliss ( 01354 653696

JUNE 2016

1 MACMILLAN CANCER SUPPORT & DOWNHAM MARKET FESTIVAL COMMITTEE Downham Market Festival Bridge (duplicate bridge with afternoon tea). Town Hall. £6 pp. 1.15 for 1.30pm. Ann Taylor ( 01366 388408

17 GREAT STUKELEY CONSERVATIVE CLUB Village Hall, Great Stukeley. £14 pp. Kay Brownlow ( 01480 880663 Sheila Stephenson ( 01480 457338

23 HUDDERSFIELD PENNINE ROTARY CLUB. Outlane Golf Club, Huddersfield. 12 for 12.30pm. £50 per table (may rise to £52) including lunch, afternoon tea & biscuits. Raffle prizes. Rtn Sam Smith ( 01924 402540 ( 07968 868828 samuelsmith396 @btinternet.com

CHARITY EVENTS

E-mail your charity events: [email protected]

JULY 2016

8 GREAT BARFORD CHURCH Village Hall, Great. Barford, Beds. £14. Derek Fordham ( 01234 870324 Gill Wilkes ( 01234 870428 [email protected]

17 CHIGWELL RIDING TRUST for people with special needs. Abridge Village Hall, Essex. 1.30 for 2pm start. £12 pp. Duplicate/Chicago available. Mrs M Walker ( 02035 390387

AUGUST 2016

19 ST IVES DAY CENTRE. £15 pp. Village Hall, Hemingford Abbots. Don Moorman ( 01480 463444

SEPTEMBER 2016

8 HUDDERSFIELD PENNINE ROTARY CLUB. 12 for 12.30pm. Outlane Golf Club, Huddersfield. £50 per table (may rise to £52) including lunch, afternoon tea & biscuits. Raffle prizes. Rtn Sam Smith ( 01924 402540 ( 07968 868828 samuelsmith396 @btinternet.com

18 CHIGWELL RIDING TRUST for people with special needs. Abridge Village Hall, Essex. 1.30 for 2pm start. £12 pp. Duplicate/Chicago available. Mrs M Walker ( 02035 390387

OCTOBER 2016

7 ST NEOTS MUSEUM. £14 pp. St Neots Outdoor Bowling Club. Jean Searle ( 01480 212298

NOVEMBER 2016

24 HUDDERSFIELD PENNINE ROTARY CLUB. 12 for 12.30pm. Outlane Golf Club, Huddersfield. £50 per table (may rise to £52) including lunch, afternoon tea & biscuits. Raffle prizes. Rtn Sam Smith ( 01924 402540 ( 07968 868828 samuelsmith396 @btinternet.com

A host of incredible experiences are packed into one truly

stunning holiday. The Caribbean, North America and Central

America: this fantastic trip takes you to some of the world’s most

beautiful, exciting and interesting destinations in a once-in-a-lifetime

adventure you will never forget.

Ponta Delgada

Madeira

Cozumel

New Orleans(2 nights)

Belize City

Banana Coast (Trujillo)

MiamiPort Canaveral

JacksonvilleHamilton

Charleston

Southampton

TortolaAntigua

BarbadosSt. Lucia

Falmouth

Tampa

Key West

Your Price Includes:Daily evening bridge on board and bridge fees included in the fare • Morning seminars and afternoon bridge on days at sea • A bridge partner will always be found for solo travellers • Breakfast, lunch, afternoon tea, a five-course a la carte dinner, late night buffet and self-service tea & coffee • Mr Bridge Welcome and Farewell Drinks parties• Varied daytime activies, lectures and talks • Choice of live evening entertainment

Prices per personCabin type Prices fromInside £3,999

Outside £4,699

Suites £10,499

Single £6,749

Prices are correct at time of going to print, but may change at any time. Call for latest prices.

mile upon mile of untouched golden beaches and two awe-inspiring, world famous rocket launch sites – the Kennedy Space Centre and Cape Canaveral Air Force Station. Contrast the fiercely modern architecture of Jacksonville with the gentler Georgian buildings of Charleston. Southern charm marks departure from the shores of the US as the British charm of Bermuda beckons.

Visit Road Town, Tortola, rich in pirate history. Here you can ride into the rugged inland landscapes on our Jeep Safari tour, or head out into the warm, crystal clear waters and swim with dolphins. Tortola was voted the best port of call in the Caribbean by Fred. Olsen guests, though Barbados, Antigua and St. Lucia aren’t bad either.

Honduras, Belize and Mexico all follow in quick succession: bathe in tropical sunshine, Latin culture and fascinating indigenous history. Little can match two nights in New Orleans though, especially as this is perfectly timed so you can join the colourful fun at the annual Mardi Gras. Collect symbolic colourful beads as the revellers (known as ‘krewes’) parade their hand-crafted floats and costumes through the city.

In Florida, enjoy Fred.’s maiden call into Tampa, followed by the party atmosphere, relaxed mind-set and beautiful beaches of Key West, Miami and Port Canaveral. This is the chance to explore floral national parks,

For adult travellers onlyThe extended length and nature of this sailing are suited to the adult traveller, therefore only those over 18 are able to join.

Key West

New Orleans

Miami, USA

Date Arrive Depart Destination

FEBRUARY 2017

Sat 04 PM Southampton

Sun 05-Tue 07 Cruising

Wed 08 early am late pm Funchal, Madeira

Thu 09-Tue 14 Cruising

Wed 15 early pm late night Bridgetown, Barbados

Thu 16 early am late pm Castries, St. Lucia

Fri 17 early am late pm St. John's, Antigua

Sat 18 early am late pm Road Town, Tortola

Sun 19-Mon 20 Cruising

Tue 21 early am late pm Falmouth, Jamaica

Wed 22 Cruising

Thu 23 early am late pm Banana Coast (Trujillo), Honduras

Fri 24 early am late pm Belize City, Belize

Sat 25 early am late pm Cozumel, Mexico

Sun 26 Cruising

Mon 27 late pm New Orleans, LA, USA

Tue 28 New Orleans, LA, USA

MARCH 2017

Wed 01 early pm New Orleans, LA, USA

Thu 02 Cruising

Fri 03 early am late night Tampa, FL, USA

Sat 04 Cruising

Sun 05 early am late pm Key West, FL, USA

Mon 06 early am late pm Miami, FL, USA

Tue 07 early am late pm Port Canaveral, FL, USAWed 08 early am late pm Jacksonville, FL, USA

Thu 09 early am late night Charleston, SC, USA

Fri 10-Sat 11 Cruising

Sun 12 early am late pm Hamilton, Bermuda

Mon 13-Fri 17 Cruising

Sat 18 early am early pm Ponta Delgada, Azores

Sun 19-Tue 21 Cruising

Wed 22 AM Southampton

REDUCED SOLE OCCUPANCY SUPPLEMENTS ON SELECTED TWIN GRADES

Up to £250 per person On Board Spend

Central America & the Mardi Gras4th February 2017 • 46 nights Sails from Southampton • Balmoral • CRUISE ID: L1703

Contact Mr Bridge to book now on 01483 489 961 or visit www.mrbridge.co.uk

FOC307429_Mr_Bridge_FP_4x_210X295_Ads_27.01.16_FV.indd 1 27/01/2016 11:38